Menu Top
Additional Questions for each Class with Solution
6th 7th 8th 9th 10th 11th 12th

Class 6th Chapters
1. Knowing Our Numbers 2. Whole Numbers 3. Playing With Numbers
4. Basic Geometrical Ideas 5. Understanding Elementary Shapes 6. Integers
7. Fractions 8. Decimals 9. Data Handling
10. Mensuration 11. Algebra 12. Ratio And Proportion
13. Symmetry 14. Practical Geometry

Content On This Page
Objective Type Questions Short Answer Type Questions Long Answer Type Questions


Chapter 8 Decimals (Additional Questions)

This section provides a valuable set of supplementary practice questions designed to reinforce and deepen your understanding of Decimals, as introduced in the Class 6th curriculum. Decimals offer another way to represent parts of a whole, closely related to fractions, and are ubiquitous in everyday life, especially in contexts involving money, measurements, and data. These additional exercises aim to provide extensive practice, ensuring you become comfortable and proficient with decimal notation, comparison, conversion, and basic arithmetic operations.

The core concepts from the chapter are revisited through these practice problems. You will strengthen your understanding of decimal place value, recognizing the value represented by digits in the tenths ($1/10$), hundredths ($1/100$), thousandths ($1/1000$), and potentially further positions to the right of the decimal point. Visualizing decimals by representing them accurately on the number line between integers will be practiced, enhancing your sense of their magnitude. A key focus is on comparing and ordering decimals, which relies heavily on a clear understanding of place value (comparing digit by digit from left to right). The crucial link between fractions and decimals is reinforced through exercises involving the conversion of fractions (especially those with denominators like 10, 100, or 1000) into their decimal equivalents and, conversely, converting decimal numbers back into fractions, often requiring simplification to the lowest terms.

Basic arithmetic operations with decimals form a significant part of the practice. You will get ample opportunity to master:

These operations will be practiced with numbers having varying numbers of decimal places, requiring careful attention to place value alignment. Furthermore, the practical application of decimals is highlighted through numerous word problems set in realistic scenarios. These often involve calculations related to:

The types of questions you can expect in this supplementary section include:

By actively engaging with these additional practice problems, you will solidify the vital connection between fractions and decimals, significantly improve your accuracy in decimal computations, deepen your understanding of the base-ten place value system as it extends to the right of the decimal point, and build a strong foundation for tackling multiplication and division of decimals, as well as their extensive use in advanced measurement and data analysis tasks.



Objective Type Questions

Question 1. In the decimal number 45.67, the digit 6 is in which place?

(A) Ones

(B) Tenths

(C) Hundredths

(D) Tens

Answer:

In the decimal number 45.67:

The digit 4 is in the Tens place.

The digit 5 is in the Ones place.

The digit 6 is in the Tenths place (the first digit after the decimal point).

The digit 7 is in the Hundredths place (the second digit after the decimal point).


Therefore, the digit 6 is in the Tenths place.

The correct option is (B) Tenths.

Question 2. The decimal representation of "Twenty-three point four five" is:

(A) 23.45

(B) 234.5

(C) 2.345

(D) 0.2345

Answer:

The phrase "Twenty-three" represents the whole number part, which is 23.

The word "point" indicates the position of the decimal point.

The phrase "four five" after the point represents the decimal part. The first digit after the decimal point is 4 (in the tenths place) and the second digit is 5 (in the hundredths place). This gives us .45.

Combining the whole number part and the decimal part, we get 23.45.


Therefore, the decimal representation of "Twenty-three point four five" is 23.45.

The correct option is (A) 23.45.

Question 3. Write the decimal number for "Seven and two tenths".

(A) 0.72

(B) 7.2

(C) 7.02

(D) 72.0

Answer:

"Seven" represents the whole number part, which is 7.

"and" indicates the decimal point.

"two tenths" means there is a 2 in the tenths place (the first digit after the decimal point). This is represented as .2.

Combining the whole number part and the decimal part, we get 7.2.


Therefore, the decimal number for "Seven and two tenths" is 7.2.

The correct option is (B) 7.2.

Question 4. How is the fraction $\frac{3}{10}$ written as a decimal?

(A) 3.0

(B) 0.3

(C) 0.03

(D) 30

Answer:

The fraction is $\frac{3}{10}$.

This fraction means 3 divided by 10.

To convert a fraction with a denominator of 10 to a decimal, we place the numerator in the tenths place.

The tenths place is the first digit to the right of the decimal point.

So, $\frac{3}{10}$ is written as 0.3.

Alternatively, performing the division $3 \div 10$ gives 0.3.


Therefore, the decimal representation of $\frac{3}{10}$ is 0.3.

The correct option is (B) 0.3.

Question 5. Convert 0.75 into a fraction in its lowest terms.

(A) $\frac{75}{100}$

(B) $\frac{3}{4}$

(C) $\frac{7}{5}$

(D) $\frac{1}{4}$

Answer:

To convert the decimal 0.75 into a fraction, we observe that there are two digits after the decimal point.

This means we can write 0.75 as a fraction with a denominator of 100 and the numerator as the digits after the decimal point.

So, $0.75 = \frac{75}{100}$.


Now, we need to convert the fraction $\frac{75}{100}$ into its lowest terms.

To do this, we find the greatest common divisor (GCD) of the numerator (75) and the denominator (100) and divide both by the GCD.

The GCD of 75 and 100 is 25.

Divide the numerator and the denominator by 25:

$\frac{75 \div 25}{100 \div 25} = \frac{3}{4}$

Using the cancellation format:

$\frac{\cancel{75}^{3}}{\cancel{100}_{4}} = \frac{3}{4}$


The fraction 3/4 cannot be simplified further as the GCD of 3 and 4 is 1.

Therefore, the decimal 0.75 converted into a fraction in its lowest terms is $\frac{3}{4}$.

The correct option is (B) $\frac{3}{4}$.

Question 6. Which of the following represents the decimal 0.4 on a number line?

Number line segment from 0 to 1, divided into 10 equal parts.

Assume the tick marks between 0 and 1 represent tenths.

(A) The 4th tick mark from 0 towards 1.

(B) The point exactly between 0 and 1.

(C) The 4th tick mark from 1 towards 0.

(D) The point slightly to the left of 0.5.

Answer:

The decimal $0.4$ represents four tenths.

As a fraction, $0.4$ is equal to $\frac{4}{10}$.

The number line segment between 0 and 1 is divided into 10 equal parts by the tick marks.

Each of these equal parts represents one tenth, or $0.1$.

Starting from 0, the first tick mark represents $0.1$, the second represents $0.2$, the third represents $0.3$, and the fourth tick mark represents $0.4$.


Therefore, the decimal $0.4$ is represented by the 4th tick mark from 0 towards 1 on the number line.

Option (B) is incorrect because the point exactly between 0 and 1 is $0.5$ or $\frac{5}{10}$.

Option (C) is incorrect because counting from 1 towards 0, the first tick mark is $0.9$, the second is $0.8$, and so on.

Option (D) is also correct in that $0.4$ is slightly to the left of $0.5$, but option (A) is a more precise description based on the tick marks representing tenths.


The correct option is (A) The 4th tick mark from 0 towards 1.

Question 7. Which pair of decimal numbers are like decimals?

(A) 1.2, 3.45

(B) 6.78, 9.01

(C) 0.5, 0.50

(D) 10.1, 10.123

Answer:

Like decimals are decimal numbers that have the same number of digits after the decimal point.


Let's examine the number of decimal places for each pair:

(A) 1.2 has one digit after the decimal point. 3.45 has two digits after the decimal point. These are not like decimals.

(B) 6.78 has two digits after the decimal point. 9.01 has two digits after the decimal point. These have the same number of digits after the decimal point.

(C) 0.5 has one digit after the decimal point. 0.50 has two digits after the decimal point. Although they represent the same value, they do not have the same number of decimal places, so they are not like decimals by definition.

(D) 10.1 has one digit after the decimal point. 10.123 has three digits after the decimal point. These are not like decimals.


The pair of decimal numbers that have the same number of digits after the decimal point is 6.78 and 9.01.

Therefore, 6.78 and 9.01 are like decimals.


The correct option is (B) 6.78, 9.01.

Question 8. To convert 8.15 into an unlike decimal with three decimal places, we write it as:

(A) 8.150

(B) 8.015

(C) 8.1500

(D) 81.50

Answer:

The given decimal number is 8.15.

This number has two digits after the decimal point (1 and 5).

We need to convert it into a decimal with three decimal places.


To increase the number of decimal places without changing the value of the decimal number, we can add zeros to the right of the last digit after the decimal point.

Adding one zero to 8.15 gives us 8.150.

The number 8.150 has three digits after the decimal point (1, 5, and 0).

The value of 8.15 is the same as 8.150, just represented with more decimal places.


Let's look at the options:

(A) 8.150: This number has three decimal places (1, 5, 0).

(B) 8.015: This number has three decimal places, but the value is different from 8.15.

(C) 8.1500: This number has four decimal places (1, 5, 0, 0).

(D) 81.50: This number has two decimal places (5, 0) and the whole number part is different.


The question asks to convert 8.15 into a decimal with three decimal places. Option (A) is the only number that has three decimal places and represents the same value as 8.15.

Therefore, to write 8.15 as an equivalent decimal with three decimal places, we write it as 8.150.


The correct option is (A) 8.150.

Question 9. Compare 0.5 and 0.50 using the correct symbol.

(A) $0.5 > 0.50$

(B) $0.5 < 0.50$

(C) $0.5 = 0.50$

(D) Cannot compare

Answer:

The decimal number 0.5 has one digit after the decimal point, which is 5 in the tenths place.

This can be written as the fraction $\frac{5}{10}$.


The decimal number 0.50 has two digits after the decimal point, which are 5 in the tenths place and 0 in the hundredths place.

This can be written as the fraction $\frac{50}{100}$.


To compare 0.5 and 0.50, we can compare their equivalent fractions $\frac{5}{10}$ and $\frac{50}{100}$.

We can simplify the fraction $\frac{50}{100}$ by dividing both the numerator and the denominator by 10:

$\frac{50}{100} = \frac{\cancel{50}^{5}}{\cancel{100}_{10}} = \frac{5}{10}$

Alternatively, we can consider the place values. The 5 in 0.5 is in the tenths place ($0.5 = 5 \times \frac{1}{10}$).

In 0.50, the 5 is in the tenths place ($5 \times \frac{1}{10}$) and the 0 is in the hundredths place ($0 \times \frac{1}{100}$).

So, $0.50 = 5 \times \frac{1}{10} + 0 \times \frac{1}{100} = \frac{5}{10} + 0 = \frac{5}{10}$.

Thus, both 0.5 and 0.50 are equivalent to $\frac{5}{10}$.

Adding zeros to the right of the last non-zero digit after the decimal point does not change the value of the decimal number.

For example, $0.5 = 0.50 = 0.500$, and so on.


Therefore, the decimal number 0.5 is equal to the decimal number 0.50.

The correct symbol to use is $=$.


The correct option is (C) $0.5 = 0.50$.

Question 10. Which of the following decimal numbers is the smallest?

(A) 1.01

(B) 1.10

(C) 1.001

(D) 1.11

Answer:

To compare the decimal numbers, we can make them like decimals by ensuring they all have the same number of digits after the decimal point. The maximum number of decimal places among the given options is three (in 1.001).

We can rewrite the numbers with three decimal places by adding trailing zeros without changing their value:

$1.01 = 1.010$

$1.10 = 1.100$

$1.001$ (already has three decimal places)

$1.11 = 1.110$


Now, we compare the numbers: 1.010, 1.100, 1.001, and 1.110.

We can compare them digit by digit from left to right, starting with the whole number part. All numbers have 1 as the whole number part.

Next, compare the tenths place:

  • 1.010 has 0 in the tenths place.
  • 1.100 has 1 in the tenths place.
  • 1.001 has 0 in the tenths place.
  • 1.110 has 1 in the tenths place.

The numbers with 0 in the tenths place (1.010 and 1.001) are smaller than the numbers with 1 in the tenths place (1.100 and 1.110).


Now, we compare the numbers that have 0 in the tenths place: 1.010 and 1.001.

Move to the hundredths place:

  • 1.010 has 1 in the hundredths place.
  • 1.001 has 0 in the hundredths place.

Since 0 is less than 1, 1.001 is smaller than 1.010.


Comparing the numbers with 1 in the tenths place (1.100 and 1.110):

Move to the hundredths place:

  • 1.100 has 0 in the hundredths place.
  • 1.110 has 1 in the hundredths place.

Since 0 is less than 1, 1.100 is smaller than 1.110.


So we have two groups: the smaller ones (1.001 and 1.010) and the larger ones (1.100 and 1.110).

The smallest among 1.001 and 1.010 is 1.001.

The smallest among all the original numbers is the smallest among the converted numbers.

Comparing 1.010, 1.100, 1.001, and 1.110, the smallest is 1.001.


Therefore, the smallest decimal number is 1.001.

The correct option is (C) 1.001.

Question 11. Calculate $12.3 + 4.56$.

(A) 16.86

(B) 16.89

(C) 16.79

(D) 16.96

Answer:

To add decimal numbers, we need to align the decimal points vertically.

It is helpful to add trailing zeros to make the numbers have the same number of decimal places (like decimals). The number 12.3 has one decimal place, and 4.56 has two decimal places. So, we can rewrite 12.3 as 12.30.

Now we add 12.30 and 4.56:

$\begin{array}{cc} & 1 & 2 & . & 3 & 0 \\ + & & 4 & . & 5 & 6 \\ \hline & 1 & 6 & . & 8 & 6 \\ \hline \end{array}$


Adding the digits in each column from right to left:

  • Hundredths column: $0 + 6 = 6$
  • Tenths column: $3 + 5 = 8$
  • Place the decimal point.
  • Ones column: $2 + 4 = 6$
  • Tens column: $1 + 0 = 1$

The sum is 16.86.


Therefore, $12.3 + 4.56 = \mathbf{16.86}$.

The correct option is (A) 16.86.

Question 12. Calculate $8.7 - 2.35$.

(A) 6.35

(B) 6.45

(C) 6.30

(D) 6.40

Answer:

To subtract decimal numbers, we need to align the decimal points vertically.

The numbers are 8.7 and 2.35.

The number 8.7 has one decimal place, while 2.35 has two decimal places.

To perform subtraction easily, it is best to make them like decimals by adding trailing zeros so that they have the same number of decimal places.

We rewrite 8.7 as 8.70 to have two decimal places.


Now we subtract 2.35 from 8.70:

Align the numbers vertically with the decimal points lined up:

$\begin{array}{cc} & 8 & . & 7 & 0 \\ - & 2 & . & 3 & 5 \\ \hline & & . & & \\ \hline \end{array}$


Perform the subtraction column by column, starting from the rightmost digit (hundredths place):

  • Hundredths column: We need to subtract 5 from 0. We cannot do this directly, so we borrow from the tenths place. The 7 in the tenths place becomes 6, and the 0 in the hundredths place becomes 10 (as we borrowed one tenth, which is ten hundredths). Now, $10 - 5 = 5$.

$\begin{array}{cc} & 8 & . & \cancel{7}^{6} & \cancel{0}^{10} \\ - & 2 & . & 3 & 5 \\ \hline & & . & & 5 \\ \hline \end{array}$

  • Tenths column: After borrowing, we have 6 in the tenths place of the top number. We subtract 3 from 6: $6 - 3 = 3$.

$\begin{array}{cc} & 8 & . & \cancel{7}^{6} & \cancel{0}^{10} \\ - & 2 & . & 3 & 5 \\ \hline & & . & 3 & 5 \\ \hline \end{array}$

  • Decimal point: Place the decimal point in the result directly below the decimal points in the numbers being subtracted.

$\begin{array}{cc} & 8 & . & \cancel{7}^{6} & \cancel{0}^{10} \\ - & 2 & . & 3 & 5 \\ \hline & & . & 3 & 5 \\ \hline \end{array}$

  • Ones column: Subtract the digits in the ones place: $8 - 2 = 6$.

$\begin{array}{cc} & 8 & . & \cancel{7}^{6} & \cancel{0}^{10} \\ - & 2 & . & 3 & 5 \\ \hline & 6 & . & 3 & 5 \\ \hline \end{array}$


The result of the subtraction is 6.35.

Therefore, $8.7 - 2.35 = \mathbf{6.35}$.


The correct option is (A) 6.35.

Question 13. Add 0.007, 8.5, and 30.08.

(A) 38.587

(B) 38.657

(C) 38.5807

(D) 38.0587

Answer:

To add decimal numbers, we need to align the decimal points vertically.

It is often helpful to make the numbers like decimals by adding trailing zeros so that they all have the same number of digits after the decimal point.

The given numbers are 0.007, 8.5, and 30.08.

The maximum number of decimal places among these numbers is three (in 0.007).

Rewrite the numbers with three decimal places:

$0.007$

$8.5 = 8.500$

$30.08 = 30.080$


Now, we arrange the numbers vertically and add, ensuring the decimal points are aligned:

$\begin{array}{ccccccc} & 0 & . & 0 & 0 & 7 \\ & 8 & . & 5 & 0 & 0 \\ + & 3 & 0 & . & 0 & 8 & 0 \\ \hline & 3 & 8 & . & 5 & 8 & 7 \\ \hline \end{array}$


Adding the digits column by column from right to left:

  • Thousandths column: $7 + 0 + 0 = 7$. Write down 7.
  • Hundredths column: $0 + 0 + 8 = 8$. Write down 8.
  • Tenths column: $0 + 5 + 0 = 5$. Write down 5.
  • Place the decimal point in the sum directly below the decimal points in the numbers being added.
  • Ones column: $0 + 8 + 0 = 8$. Write down 8.
  • Tens column: $0 + 0 + 3 = 3$. Write down 3.

The sum obtained is 38.587.


Therefore, $0.007 + 8.5 + 30.08 = \mathbf{38.587}$.

The correct option is (A) 38.587.

Question 14. A cyclist covers a distance of 15.75 km in the first hour and 12.5 km in the second hour. What is the total distance covered?

(A) 27.25 km

(B) 28.25 km

(C) 28.75 km

(D) 3.25 km

Answer:

Given:

Distance covered in the first hour = 15.75 km

Distance covered in the second hour = 12.5 km


To Find:

Total distance covered.


Solution:

The total distance covered is the sum of the distance covered in the first hour and the distance covered in the second hour.

Total distance = Distance in first hour + Distance in second hour

Total distance $= 15.75 \text{ km} + 12.5 \text{ km}$


To add decimal numbers, we need to align the decimal points and add column by column.

We can rewrite 12.5 as 12.50 to have the same number of decimal places as 15.75.

$\begin{array}{ccccccc} & 1 & 5 & . & 7 & 5 \\ + & 1 & 2 & . & 5 & 0 \\ \hline & 2 & 8 & . & 2 & 5 \\ \hline \end{array}$


Adding the digits from right to left:

  • Hundredths place: $5 + 0 = 5$.
  • Tenths place: $7 + 5 = 12$. Write down 2 and carry over 1 to the ones place.
  • Place the decimal point.
  • Ones place: $5 + 2 + 1$ (carry over) $= 8$.
  • Tens place: $1 + 1 = 2$.

The sum is 28.25.


The total distance covered by the cyclist is $\mathbf{28.25}$ km.

The correct option is (B) 28.25 km.

Question 15. Raghav bought vegetables weighing 10 kg. Out of this, 3 kg 500 g is onions, 2 kg 75 g is tomatoes, and the rest is potatoes. What is the weight of the potatoes?

(A) 4.425 kg

(B) 4.525 kg

(C) 4.400 kg

(D) 4.325 kg

Answer:

Given:

Total weight of vegetables = 10 kg

Weight of onions = 3 kg 500 g

Weight of tomatoes = 2 kg 75 g

The rest is potatoes.


To Find:

The weight of the potatoes.


Solution:

First, we convert the weights of onions and tomatoes into kilograms using decimals.

We know that $1 \text{ kg} = 1000 \text{ g}$.

So, $500 \text{ g} = \frac{500}{1000} \text{ kg} = 0.500 \text{ kg}$.

Weight of onions = $3 \text{ kg} + 0.500 \text{ kg} = 3.500 \text{ kg}$.


$75 \text{ g} = \frac{75}{1000} \text{ kg} = 0.075 \text{ kg}$.

Weight of tomatoes = $2 \text{ kg} + 0.075 \text{ kg} = 2.075 \text{ kg}$.


The total weight of onions and tomatoes is the sum of their individual weights:

Weight of onions and tomatoes = Weight of onions + Weight of tomatoes

Weight of onions and tomatoes = $3.500 \text{ kg} + 2.075 \text{ kg}$


Adding 3.500 and 2.075:

$\begin{array}{ccccccc} & 3 & . & 5 & 0 & 0 \\ + & 2 & . & 0 & 7 & 5 \\ \hline & 5 & . & 5 & 7 & 5 \\ \hline \end{array}$

The combined weight of onions and tomatoes is 5.575 kg.


The total weight of vegetables is 10 kg. This total weight is the sum of the weights of onions, tomatoes, and potatoes.

Total weight = Weight of onions and tomatoes + Weight of potatoes

$10 \text{ kg} = 5.575 \text{ kg} + \text{Weight of potatoes}$


To find the weight of the potatoes, we subtract the combined weight of onions and tomatoes from the total weight.

Weight of potatoes = Total weight - Weight of onions and tomatoes

Weight of potatoes $= 10 \text{ kg} - 5.575 \text{ kg}$


To subtract 5.575 from 10, we can write 10 as 10.000 to have the same number of decimal places.

Subtracting 5.575 from 10.000:

$\begin{array}{ccccccc} & \cancel{1}^{0} & \cancel{0}^{9} & . & \cancel{0}^{9} & \cancel{0}^{9} & \cancel{0}^{10} \\ - & & 5 & . & 5 & 7 & 5 \\ \hline & & 4 & . & 4 & 2 & 5 \\ \hline \end{array}$

The difference is 4.425.


The weight of the potatoes is $\mathbf{4.425}$ kg.

The correct option is (A) 4.425 kg.

Question 16. Write 2 metres and 35 centimetres in metres using decimals.

(A) 2.35 m

(B) 2.035 m

(C) 2.350 m

(D) 23.5 m

Answer:

Given:

Length = 2 metres and 35 centimetres.


To Find:

The length in metres using decimals.


Solution:

We need to convert the given length entirely into metres.

We know the relationship between metres and centimetres:

$1 \text{ metre} = 100 \text{ centimetres}$

To convert centimetres to metres, we divide the number of centimetres by 100.

The centimetres part of the given length is 35 centimetres.

Converting 35 centimetres to metres:

$35 \text{ centimetres} = \frac{35}{100} \text{ metres}$


Now, we write the fraction $\frac{35}{100}$ as a decimal.

$\frac{35}{100} = 0.35$ metres


The total length is the sum of the whole metres part and the decimal metres part obtained from the centimetres.

Total length = 2 metres + 35 centimetres

Total length = 2 metres + 0.35 metres

Total length $= 2.35$ metres.


Therefore, 2 metres and 35 centimetres is equal to 2.35 m.

The correct option is (A) 2.35 m.

Question 17. Write 50 paise in rupees using decimals.

(A) $\textsf{₹}$ 50.00

(B) $\textsf{₹}$ 0.50

(C) $\textsf{₹}$ 5.00

(D) $\textsf{₹}$ 0.05

Answer:

Given:

Amount in paise = 50 paise.


To Find:

The amount in rupees using decimals.


Solution:

We know the relationship between Rupees and Paise:

$1 \text{ Rupee} = 100 \text{ Paise}$

To convert a value from paise to rupees, we need to divide the amount in paise by 100.

Amount in rupees = $\frac{\text{Amount in paise}}{100}$


In this case, the amount is 50 paise.

Amount in rupees = $\frac{50}{100}$

To write the fraction $\frac{50}{100}$ as a decimal, we divide 50 by 100.

Dividing by 100 means moving the decimal point two places to the left.

$50.0 \div 100 = 0.50$

So, $\frac{50}{100} = 0.50$ rupees.


Therefore, 50 paise written in rupees using decimals is $\textsf{₹}$ 0.50.

The correct option is (B) $\textsf{₹}$ 0.50.

Question 18. Which of the following is equivalent to 0.8?

(A) $\frac{8}{100}$

(B) $\frac{4}{5}$

(C) $0.08$

(D) $\frac{80}{10}$

Answer:

The decimal number given is $0.8$.

The digit 8 is in the tenths place.

This means that $0.8$ can be written as the fraction $\frac{8}{10}$.


Now, we need to check which of the given options is equivalent to $\frac{8}{10}$.

Let's simplify the fraction $\frac{8}{10}$ to its lowest terms by dividing the numerator and the denominator by their greatest common divisor, which is 2.

$\frac{8}{10} = \frac{\cancel{8}^{4}}{\cancel{10}_{5}} = \frac{4}{5}$


Now let's evaluate each option:

(A) $\frac{8}{100}$: As a decimal, this is $0.08$. $0.08$ is not equal to $0.8$.

(B) $\frac{4}{5}$: We found that $\frac{8}{10}$ simplifies to $\frac{4}{5}$. So, $0.8$ is equivalent to $\frac{4}{5}$.

We can also convert $\frac{4}{5}$ to a decimal by dividing 4 by 5, or by converting the denominator to 10:

$\frac{4}{5} = \frac{4 \times 2}{5 \times 2} = \frac{8}{10} = 0.8$.

(C) $0.08$: This decimal has the digit 8 in the hundredths place. It is not equal to $0.8$.

(D) $\frac{80}{10}$: This fraction simplifies to $80 \div 10 = 8$. $8$ is not equal to $0.8$.


Comparing $0.8$ with the values of the options, we find that $\frac{4}{5}$ is equivalent to $0.8$.


The correct option is (B) $\frac{4}{5}$.

Question 19. Match the decimal with its value:

(i) 0.2

(ii) 0.02

(iii) 2.0

(iv) 0.20

(a) Two hundredths

(b) Two

(c) Two tenths

(d) Equivalent to 0.2

(A) (i)-(c), (ii)-(a), (iii)-(b), (iv)-(d)

(B) (i)-(c), (ii)-(a), (iii)-(d), (iv)-(b)

(C) (i)-(a), (ii)-(c), (iii)-(b), (iv)-(d)

(D) (i)-(c), (ii)-(b), (iii)-(a), (iv)-(d)

Answer:

Let's determine the value of each decimal number:

  • (i) $0.2$: The digit 2 is in the first place after the decimal point, which is the tenths place. So, 0.2 represents Two tenths.
  • (ii) $0.02$: The digit 2 is in the second place after the decimal point, which is the hundredths place. So, 0.02 represents Two hundredths.
  • (iii) $2.0$: The digit 2 is in the ones place before the decimal point. The digit after the decimal is 0 in the tenths place. So, 2.0 represents Two.
  • (iv) $0.20$: The digit 2 is in the tenths place and 0 is in the hundredths place. Adding a zero at the end of a decimal number does not change its value. So, 0.20 is Equivalent to 0.2.

Matching the decimals with their values/descriptions:

  • (i) 0.2 matches with (c) Two tenths.
  • (ii) 0.02 matches with (a) Two hundredths.
  • (iii) 2.0 matches with (b) Two.
  • (iv) 0.20 matches with (d) Equivalent to 0.2.

The correct matching is (i)-(c), (ii)-(a), (iii)-(b), (iv)-(d).

Comparing this with the given options, we find that Option (A) matches this set of pairs.


The correct option is (A) (i)-(c), (ii)-(a), (iii)-(b), (iv)-(d).

Question 20. Assertion (A): 1.5 is greater than 1.45.
Reason (R): To compare decimals, we compare the digits from left to right. If the whole number parts are equal, we compare the tenths digit, then the hundredths, and so on.

(A) Both A and R are true and R is the correct explanation of A.

(B) Both A and R are true but R is not the correct explanation of A.

(C) A is true but R is false.

(D) A is false but R is true.

Answer:

Let's analyze the given Assertion (A) and Reason (R).


Assertion (A): 1.5 is greater than 1.45.

To compare 1.5 and 1.45, we can write them as like decimals. 1.5 has one decimal place, and 1.45 has two decimal places. We can add a zero to 1.5 to make it 1.50.

Now we compare 1.50 and 1.45.

Compare the whole number parts: $1 = 1$.

Compare the digits in the tenths place: For 1.50, the digit is 5. For 1.45, the digit is 4.

Since $5 > 4$, the number with the larger digit in the tenths place is greater.

So, $1.50 > 1.45$. This means $1.5 > 1.45$.

Thus, Assertion (A) is True.


Reason (R): To compare decimals, we compare the digits from left to right. If the whole number parts are equal, we compare the tenths digit, then the hundredths, and so on.

This statement accurately describes the standard procedure for comparing decimal numbers. We always start comparing from the leftmost digit (highest place value) and move to the right. If the whole number parts are different, the number with the larger whole number part is greater. If the whole number parts are the same, we move to the tenths place and compare the digits. If the tenths digits are different, the number with the larger tenths digit is greater. If they are the same, we move to the hundredths place and continue this process until we find a difference or run out of digits (in which case the numbers are equal if made into like decimals).

Thus, Reason (R) is True.


Now we check if Reason (R) is the correct explanation for Assertion (A).

Assertion (A) states that 1.5 > 1.45. The method we used to verify this fact in the analysis of Assertion (A) was exactly the method described in Reason (R) - comparing digits from left to right, starting with the whole number part, then the tenths place, etc.

Therefore, Reason (R) provides the correct method and explanation for why Assertion (A) is true.


Both Assertion (A) and Reason (R) are true, and Reason (R) is the correct explanation for Assertion (A).

Based on this analysis, the correct option is (A).


The correct option is (A) Both A and R are true and R is the correct explanation of A.

Question 21. Which of the following operations is correctly solved?

(A) $5.2 + 3.1 = 8.3$

(B) $7.5 - 2.1 = 5.4$

(C) $0.9 + 0.01 = 0.10$

(D) $10.0 - 0.5 = 9.0$

Answer:

We need to evaluate each operation to check if the given equation is correctly solved.


Option (A): $5.2 + 3.1$

Let's perform the addition:

$\begin{array}{cc} & 5 & . & 2 \\ + & 3 & . & 1 \\ \hline & 8 & . & 3 \\ \hline \end{array}$

The sum is 8.3.

The given equation is $5.2 + 3.1 = 8.3$. This is a Correct statement.


Option (B): $7.5 - 2.1$

Let's perform the subtraction:

$\begin{array}{cc} & 7 & . & 5 \\ - & 2 & . & 1 \\ \hline & 5 & . & 4 \\ \hline \end{array}$

The difference is 5.4.

The given equation is $7.5 - 2.1 = 5.4$. This is a Correct statement.


Option (C): $0.9 + 0.01$

Make the numbers like decimals by writing 0.9 as 0.90.

Let's perform the addition:

$\begin{array}{cc} & 0 & . & 9 & 0 \\ + & 0 & . & 0 & 1 \\ \hline & 0 & . & 9 & 1 \\ \hline \end{array}$

The sum is 0.91.

The given equation is $0.9 + 0.01 = 0.10$. Since $0.91 \neq 0.10$, this is an Incorrect statement.


Option (D): $10.0 - 0.5$

Let's perform the subtraction:

$\begin{array}{cc} & 1 & 0 & . & 0 \\ - & & 0 & . & 5 \\ \hline & & 9 & . & 5 \\ \hline \end{array}$

The difference is 9.5.

The given equation is $10.0 - 0.5 = 9.0$. Since $9.5 \neq 9.0$, this is an Incorrect statement.


Based on the calculations, both Option (A) and Option (B) show operations that are correctly solved.

In a standard objective type question format where only one option is correct, there might be an error in the question as both (A) and (B) are mathematically sound.

However, if we must choose one from the given options, and assuming a unique correct answer is intended, let's re-examine. Both calculations are straightforward. Without further context or clarification on the question's intent or possible subtleties (which are not apparent), we will state that both (A) and (B) are correct based on direct computation.

If only one answer is allowed, we will select (A) as the listed correct option based on common question construction, although (B) is also correct.

The correct option is (A) $5.2 + 3.1 = 8.3$.

Question 22. Convert 150 grams into kilograms using decimals.

(A) 150.0 kg

(B) 1.50 kg

(C) 0.150 kg

(D) 0.0150 kg

Answer:

Given:

Weight = 150 grams.


To Find:

The weight in kilograms using decimals.


Solution:

We need to convert grams to kilograms.

The relationship between kilograms and grams is:

$1 \text{ kilogram (kg)} = 1000 \text{ grams (g)}$

To convert grams to kilograms, we divide the number of grams by 1000.

Weight in kg = $\frac{\text{Weight in grams}}{1000}$


Given weight is 150 grams.

Weight in kg = $\frac{150}{1000} \text{ kg}$


To write the fraction $\frac{150}{1000}$ as a decimal, we divide 150 by 1000.

Dividing by 1000 means moving the decimal point three places to the left from its original position (after the last digit).

$150.0 \div 1000 = 0.150$

So, $\frac{150}{1000} = 0.150$ kg.


Therefore, 150 grams is equal to 0.150 kg.

The correct option is (C) 0.150 kg.

Question 23. What is the value of $25.5 - 10.25 + 3.7$?

(A) 19.95

(B) 18.95

(C) 18.05

(D) 19.05

Answer:

We need to evaluate the expression $25.5 - 10.25 + 3.7$.

We can perform the operations from left to right.


First, calculate $25.5 - 10.25$.

To subtract, align the decimal points and add trailing zeros to make them like decimals. 25.5 has one decimal place, and 10.25 has two. Rewrite 25.5 as 25.50.

Subtract 10.25 from 25.50:

$\begin{array}{ccccccc} & 2 & 5 & . & 5 & 0 \\ - & 1 & 0 & . & 2 & 5 \\ \hline & 1 & 5 & . & 2 & 5 \\ \hline \end{array}$

So, $25.5 - 10.25 = 15.25$.


Next, add 3.7 to the result, 15.25.

To add, align the decimal points and add trailing zeros. 15.25 has two decimal places, and 3.7 has one. Rewrite 3.7 as 3.70.

Add 15.25 and 3.70:

$\begin{array}{ccccccc} & 1 & 5 & . & 2 & 5 \\ + & & 3 & . & 7 & 0 \\ \hline & 1 & 8 & . & 9 & 5 \\ \hline \end{array}$

The sum is 18.95.


Therefore, the value of $25.5 - 10.25 + 3.7$ is $\mathbf{18.95}$.

The correct option is (B) 18.95.

Question 24. Which of the following is NOT a correct representation of place value in decimals?

(A) Tenths = $\frac{1}{10}$

(B) Hundredths = $\frac{1}{100}$

(C) Thousandths = $\frac{1}{1000}$

(D) Ten-tenths = $\frac{1}{100}$

Answer:

In decimal numbers, the place value of digits to the right of the decimal point are fractions with denominators that are powers of 10.

  • The first place after the decimal point is the tenths place. Its value is $\frac{1}{10}$.
  • The second place after the decimal point is the hundredths place. Its value is $\frac{1}{100}$.
  • The third place after the decimal point is the thousandths place. Its value is $\frac{1}{1000}$.

Let's examine each option:

  • (A) Tenths = $\frac{1}{10}$: This matches the definition of the tenths place value. This is a correct representation.
  • (B) Hundredths = $\frac{1}{100}$: This matches the definition of the hundredths place value. This is a correct representation.
  • (C) Thousandths = $\frac{1}{1000}$: This matches the definition of the thousandths place value. This is a correct representation.
  • (D) Ten-tenths = $\frac{1}{100}$:

    "Ten-tenths" means $10 \times \text{tenths}$.

    $10 \times \frac{1}{10} = \frac{10}{10} = 1$.

    So, "Ten-tenths" is equal to 1.

    The statement says "Ten-tenths = $\frac{1}{100}$".

    Since $1 \neq \frac{1}{100}$, this is an incorrect representation.

    The value $\frac{1}{100}$ represents "one hundredth", not "ten tenths".


The representation that is NOT correct is Ten-tenths = $\frac{1}{100}$.

The correct option is (D) Ten-tenths = $\frac{1}{100}$.

Question 25. Case Study: A shopkeeper sold 12.5 litres of milk on Monday, 15.75 litres on Tuesday, and 18 litres on Wednesday. He started with 50 litres of milk.

How many litres of milk are left with him at the end of Wednesday?

(A) 36.25 litres

(B) 13.75 litres

(C) 14.25 litres

(D) 12.75 litres

Answer:

Given:

Initial quantity of milk = 50 litres

Milk sold on Monday = 12.5 litres

Milk sold on Tuesday = 15.75 litres

Milk sold on Wednesday = 18 litres


To Find:

The quantity of milk left at the end of Wednesday.


Solution:

First, calculate the total quantity of milk sold over the three days.

Total milk sold = Milk sold on Monday + Milk sold on Tuesday + Milk sold on Wednesday

Total milk sold = $12.5 + 15.75 + 18$ litres

To perform the addition of decimal numbers, we should align the decimal points. It is helpful to add trailing zeros to make the numbers have the same number of decimal places. The number 15.75 has two decimal places, while 12.5 has one and 18 has zero. We can rewrite them with two decimal places as 12.50, 15.75, and 18.00.

Total milk sold = $12.50 + 15.75 + 18.00$ litres

We can add these numbers vertically:

$\begin{array}{ccccccc} & 1 & 2 & . & 5 & 0 \\ & 1 & 5 & . & 7 & 5 \\ + & 1 & 8 & . & 0 & 0 \\ \hline & 4 & 6 & . & 2 & 5 \\ \hline \end{array}$

So, the total quantity of milk sold over the three days is 46.25 litres.


Next, subtract the total quantity of milk sold from the initial quantity to find the quantity of milk left.

Milk left = Initial quantity - Total milk sold

Milk left = $50 - 46.25$ litres

To perform the subtraction, write 50 as 50.00 and align the decimal points:

$\begin{array}{ccccccc} & 5 & 0 & . & 0 & 0 \\ - & 4 & 6 & . & 2 & 5 \\ \hline & & 3 & . & 7 & 5 \\ \hline \end{array}$

The calculated amount of milk left is 3.75 litres.


Based on the calculations using the given values, the amount of milk left is 3.75 litres. However, this result is not among the provided options.

Assuming there might be a discrepancy in the question or options and that one of the options is intended to be correct, we provide the option corresponding to a likely intended problem variation (where Wednesday's sale was intended to be 8 litres instead of 18, resulting in a total sale of 36.25 litres and a remainder of 13.75 litres).

The correct option is (B) 13.75 litres.

Question 26. Arrange the following decimals in ascending order: $2.3, 2.03, 2.30, 2.003$.

(A) $2.3, 2.03, 2.30, 2.003$

(B) $2.003, 2.03, 2.3, 2.30$

(C) $2.03, 2.003, 2.3, 2.30$

(D) $2.003, 2.03, 2.30, 2.3$

Answer:

Given decimals: 2.3, 2.03, 2.30, 2.003.


To Arrange: In ascending order (from smallest to largest).


Solution:

To compare and arrange decimal numbers, it is helpful to make them like decimals by adding trailing zeros so that they all have the same number of digits after the decimal point.

The given numbers are:

  • 2.3 (one decimal place)
  • 2.03 (two decimal places)
  • 2.30 (two decimal places)
  • 2.003 (three decimal places)

The maximum number of decimal places is three. Let's rewrite all numbers with three decimal places:

  • $2.3 = 2.300$
  • $2.03 = 2.030$
  • $2.30 = 2.300$
  • $2.003$ (already has three decimal places)

Now we compare the numbers 2.300, 2.030, 2.300, and 2.003 digit by digit from left to right:

1. Compare the whole number parts: All are 2. They are equal.

2. Compare the tenths digits: The tenths digits are 3 (in 2.300), 0 (in 2.030), 3 (in 2.300), and 0 (in 2.003). The smallest tenths digit is 0.

So, the smallest numbers are among 2.030 and 2.003.

3. Compare the hundredths digits for 2.030 and 2.003: The hundredths digits are 3 (in 2.030) and 0 (in 2.003). Since $0 < 3$, the number 2.003 is smaller than 2.030.

Thus, 2.003 is the smallest, followed by 2.030.

4. The remaining numbers are 2.300 and 2.300. They are equal.


Arranging the converted numbers in ascending order:

$2.003, 2.030, 2.300, 2.300$


Replacing them with the original numbers:

The ascending order is $2.003, 2.03, 2.3, 2.30$.

Note that $2.3$ and $2.30$ are equivalent, so their relative order in the list can be either way after $2.03$, but both must come after $2.03$.


Checking the options:

  • (A) $2.3, 2.03, 2.30, 2.003$ (Incorrect)
  • (B) $2.003, 2.03, 2.3, 2.30$ (Correct)
  • (C) $2.03, 2.003, 2.3, 2.30$ (Incorrect, 2.03 is not smaller than 2.003)
  • (D) $2.003, 2.03, 2.30, 2.3$ (Correct, as 2.30 and 2.3 are equivalent)

Both options (B) and (D) represent the correct ascending order. Assuming there is a unique correct answer intended, and given the order of 2.3 and 2.30 in the original list, option (B) which lists 2.3 before 2.30 is the likely intended answer.


The correct option is (B) $2.003, 2.03, 2.3, 2.30$.

Question 27. Which of the following decimals lie between 3 and 4 on the number line? (Select all that apply)

(A) 3.5

(B) 4.1

(C) 3.05

(D) 2.9

Answer:

A decimal number lies between 3 and 4 on the number line if it is greater than 3 and less than 4.

In mathematical terms, a number $x$ is between 3 and 4 if $3 < x < 4$.


Let's check each given option:

  • (A) 3.5: Is $3 < 3.5 < 4$? Yes, 3.5 is greater than 3 and less than 4. So, 3.5 lies between 3 and 4.
  • (B) 4.1: Is $3 < 4.1 < 4$? No, 4.1 is greater than 4. So, 4.1 does not lie between 3 and 4.
  • (C) 3.05: Is $3 < 3.05 < 4$? Yes, 3.05 is greater than 3 (since 3.05 has 0 in the tenths place and 5 in the hundredths place, while 3 can be thought of as 3.00) and less than 4. So, 3.05 lies between 3 and 4.
  • (D) 2.9: Is $3 < 2.9 < 4$? No, 2.9 is less than 3. So, 2.9 does not lie between 3 and 4.

The decimal numbers that lie between 3 and 4 are 3.5 and 3.05.


The correct options are (A) 3.5 and (C) 3.05.

Question 28. Write the decimal number for $50 + 6 + \frac{2}{10} + \frac{9}{1000}$.

(A) 56.29

(B) 56.209

(C) 56.029

(D) 56.290

Answer:

Given expression: $50 + 6 + \frac{2}{10} + \frac{9}{1000}$.


To Find:

The decimal number for the given expression.


Solution:

The expression represents the sum of a whole number part and fractional parts.

The whole number part is the sum of the terms that are not fractions:

Whole number part $= 50 + 6 = 56$.


The fractional parts are given as fractions with denominators 10 and 1000. These correspond to the decimal places.

  • $\frac{2}{10}$ represents 2 tenths. This corresponds to the first decimal place, which is the tenths place. As a decimal, $\frac{2}{10} = 0.2$.
  • $\frac{9}{1000}$ represents 9 thousandths. This corresponds to the third decimal place, which is the thousandths place. As a decimal, $\frac{9}{1000} = 0.009$.

There is no term with a denominator of 100, which corresponds to the hundredths place. The digit in the hundredths place will be 0.


The decimal number is formed by combining the whole number part and the decimal parts corresponding to the fractional terms.

The whole number part is 56.

The tenths digit comes from $\frac{2}{10}$, which is 2.

The hundredths digit corresponds to $\frac{\text{something}}{100}$. Since there is no such term, the digit is 0.

The thousandths digit comes from $\frac{9}{1000}$, which is 9.

Combining these, the decimal number is 56.209.


Alternatively, we can add the decimal values:

$50$ as a decimal is $50.000$

$6$ as a decimal is $6.000$

$\frac{2}{10}$ as a decimal is $0.2$, which is $0.200$ with three decimal places.

$\frac{9}{1000}$ as a decimal is $0.009$.

Adding these values vertically, aligning the decimal points:

$\begin{array}{ccccccc} & 5 & 0 & . & 0 & 0 & 0 \\ & & 6 & . & 0 & 0 & 0 \\ & & 0 & . & 2 & 0 & 0 \\ + & & 0 & . & 0 & 0 & 9 \\ \hline & 5 & 6 & . & 2 & 0 & 9 \\ \hline \end{array}$

The sum is 56.209.


Therefore, the decimal number for $50 + 6 + \frac{2}{10} + \frac{9}{1000}$ is $\mathbf{56.209}$.

The correct option is (B) 56.209.

Question 29. Complete the sentence: Adding zeros to the extreme right of the decimal part of a number _____ its value.

(A) Increases

(B) Decreases

(C) Does not change

(D) Depends on the digits

Answer:

Let's consider an example to understand the effect of adding zeros to the extreme right of the decimal part of a number.

Consider the decimal number 0.5.

As a fraction, 0.5 is $\frac{5}{10}$.

If we add one zero to the extreme right, we get 0.50.

As a fraction, 0.50 is $\frac{50}{100}$.

Simplifying the fraction $\frac{50}{100}$: $\frac{\cancel{50}^{1}}{\cancel{100}_{2}} = \frac{1}{2}$.

Simplifying the fraction $\frac{5}{10}$: $\frac{\cancel{5}^{1}}{\cancel{10}_{2}} = \frac{1}{2}$.

Both fractions are equal to $\frac{1}{2}$, which is 0.5.


Consider another example: 1.23.

As a fraction, 1.23 is $1 + \frac{23}{100} = \frac{123}{100}$.

If we add a zero, we get 1.230.

As a fraction, 1.230 is $1 + \frac{230}{1000} = \frac{1230}{1000}$.

Simplifying $\frac{1230}{1000}$ by dividing the numerator and denominator by 10 gives $\frac{123}{100}$.

So, $1.23 = 1.230$.


Adding zeros to the extreme right of the decimal part of a number is equivalent to multiplying the numerator and the denominator of the corresponding fraction by a power of 10 (like 10, 100, etc.), which does not change the value of the fraction.

For example, $0.2 = \frac{2}{10}$. Adding a zero gives $0.20 = \frac{20}{100}$. We know that $\frac{2}{10} = \frac{2 \times 10}{10 \times 10} = \frac{20}{100}$.


Therefore, adding zeros to the extreme right of the decimal part of a number does not change its value.

The correct option is (C) Does not change.

Question 30. If you have $\textsf{₹}$ 20 and you spend $\textsf{₹}$ 12.75, how much money do you have left?

(A) $\textsf{₹}$ 7.25

(B) $\textsf{₹}$ 8.25

(C) $\textsf{₹}$ 7.75

(D) $\textsf{₹}$ 8.75

Answer:

Given:

Initial amount of money = $\textsf{₹}$ 20

Amount spent = $\textsf{₹}$ 12.75


To Find:

The amount of money left.


Solution:

To find the amount of money left, we need to subtract the amount spent from the initial amount.

Amount left = Initial amount - Amount spent

Amount left $= \textsf{₹} 20 - \textsf{₹} 12.75$


To perform subtraction with decimals, we must align the decimal points. We can write $\textsf{₹}$ 20 as $\textsf{₹}$ 20.00 to match the number of decimal places in $\textsf{₹}$ 12.75.

Subtract $\textsf{₹}$ 12.75 from $\textsf{₹}$ 20.00:

$\begin{array}{ccccccc} & 2 & 0 & . & 0 & 0 \\ - & 1 & 2 & . & 7 & 5 \\ \hline & & & . & & \\ \hline \end{array}$

We perform the subtraction column by column, starting from the rightmost digit (hundredths place). We need to borrow from the left.

$\begin{array}{ccccccc} & \cancel{2}^{1} & \cancel{0}^{9} & . & \cancel{0}^{9} & \cancel{0}^{10} \\ - & & 1 & 2 & . & 7 & 5 \\ \hline & & & 7 & . & 2 & 5 \\ \hline \end{array}$

The result of the subtraction is 7.25.


So, the amount of money left is $\mathbf{\textsf{₹} 7.25}$.

The correct option is (A) $\textsf{₹}$ 7.25.

Question 31. The decimal form of $\frac{1}{2}$ is:

(A) 0.12

(B) 0.2

(C) 0.5

(D) 0.25

Answer:

To convert a fraction to a decimal, we divide the numerator by the denominator.

The given fraction is $\frac{1}{2}$.

We need to calculate $1 \div 2$.


Performing the division:

$1 \div 2 = 0.5$


Alternatively, we can convert the fraction to an equivalent fraction with a denominator that is a power of 10 (like 10, 100, 1000, etc.).

We can convert the denominator 2 to 10 by multiplying both the numerator and the denominator by 5:

$\frac{1}{2} = \frac{1 \times 5}{2 \times 5} = \frac{5}{10}$

Now, we write the fraction $\frac{5}{10}$ as a decimal. The denominator is 10, so the numerator goes in the tenths place (the first digit after the decimal point).

$\frac{5}{10} = 0.5$


Thus, the decimal form of $\frac{1}{2}$ is 0.5.

The correct option is (C) 0.5.

Question 32. Which is greater: 2.0 or 1.99?

(A) 2.0

(B) 1.99

(C) They are equal

(D) Cannot compare

Answer:

To compare the decimal numbers 2.0 and 1.99, we can make them like decimals by adding trailing zeros so that they have the same number of decimal places. The number 1.99 has two decimal places, while 2.0 has one.

Rewrite 2.0 with two decimal places: $2.0 = 2.00$.

Now we compare 2.00 and 1.99.


We compare the numbers digit by digit from left to right, starting with the whole number part:

  • Compare the whole number parts: For 2.00, the whole number part is 2. For 1.99, the whole number part is 1.
  • Since $2 > 1$, the number with the larger whole number part is greater.

So, $2.00 > 1.99$, which means $2.0 > 1.99$.


Alternatively, we can think about the number line. The number 2.0 is exactly at 2. The number 1.99 is very close to 2 but is less than 2. On the number line, 1.99 is to the left of 2.0.


Therefore, 2.0 is greater than 1.99.

The correct option is (A) 2.0.

Question 33. What is the result of $10.5 - 4.8$? (Completion Question)

Result: _____

(A) 5.7

(B) 6.7

(C) 5.3

(D) 6.3

Answer:

Given expression: $10.5 - 4.8$


To Find:

The result of the subtraction.


Solution:

To subtract decimal numbers, we align the decimal points vertically.

The numbers are 10.5 and 4.8. Both have one decimal place, so they are already like decimals.

Subtract 4.8 from 10.5:

$\begin{array}{ccccccc} & 1 & 0 & . & 5 \\ - & & 4 & . & 8 \\ \hline & & & . & \\ \hline \end{array}$

Subtract column by column from right to left:

  • Tenths place: We need to subtract 8 from 5. We cannot do this directly, so we borrow from the ones place. The 0 in the ones place becomes 9 (after borrowing from the tens place), and the 5 in the tenths place becomes 15. Now, $15 - 8 = 7$.

$\begin{array}{ccccccc} & \cancel{1}^{0} & \cancel{0}^{9} & . & \cancel{5}^{15} \\ - & & & 4 & . & 8 \\ \hline & & & . & 7 \\ \hline \end{array}$

  • Ones place: After borrowing, we have 9 in the ones place of the top number. We subtract 4 from 9: $9 - 4 = 5$.

$\begin{array}{ccccccc} & \cancel{1}^{0} & \cancel{0}^{9} & . & \cancel{5}^{15} \\ - & & & 4 & . & 8 \\ \hline & & 5 & . & 7 \\ \hline \end{array}$

  • Tens place: We have 0 in the tens place after borrowing. Subtract 0 (from 4.8) from 0: $0 - 0 = 0$.

$\begin{array}{ccccccc} & \cancel{1}^{0} & \cancel{0}^{9} & . & \cancel{5}^{15} \\ - & & & 4 & . & 8 \\ \hline & & 5 & . & 7 \\ \hline \end{array}$

The difference is 5.7.


The result of $10.5 - 4.8$ is $\mathbf{5.7}$.

Comparing this result with the options, we find that option (A) matches the calculated result.


The correct option is (A) 5.7.

Question 34. Identify the unlike decimal from the following set: 3.14, 5.2, 0.75, 12.00.

(A) 3.14

(B) 5.2

(C) 0.75

(D) All are unlike decimals

Answer:

Like decimals are decimal numbers that have the same number of digits after the decimal point.

Unlike decimals are decimal numbers that have a different number of digits after the decimal point.


Let's count the number of decimal places for each number in the given set: 3.14, 5.2, 0.75, 12.00.

  • 3.14 has two digits after the decimal point (1 and 4).
  • 5.2 has one digit after the decimal point (2).
  • 0.75 has two digits after the decimal point (7 and 5).
  • 12.00 has two digits after the decimal point (0 and 0).

We are looking for the decimal number that has a different number of decimal places compared to the others.

The numbers 3.14, 0.75, and 12.00 all have two decimal places. They are like decimals among themselves.

The number 5.2 has only one decimal place.

Therefore, 5.2 is the unlike decimal in this set because it has a different number of decimal places than the other numbers.


The correct option is (B) 5.2.

Question 35. What is the place value of the digit 3 in 15.038?

(A) Tenths

(B) Hundredths

(C) Thousandths

(D) Ones

Answer:

In the decimal number 15.038, the digits have place values relative to the decimal point.

  • Digits to the left of the decimal point represent whole number place values:
    • 5 is in the Ones place.
    • 1 is in the Tens place.

  • Digits to the right of the decimal point represent fractional place values:
    • The first digit after the decimal point (0) is in the Tenths place, which has a value of $\frac{1}{10}$.
    • The second digit after the decimal point (3) is in the Hundredths place, which has a value of $\frac{1}{100}$.
    • The third digit after the decimal point (8) is in the Thousandths place, which has a value of $\frac{1}{1000}$.

The digit 3 is the second digit to the right of the decimal point.

Its place value is the Hundredths place.


The correct option is (B) Hundredths.



Short Answer Type Questions

Question 1. Write the decimal number for "two and five tenths".

Answer:

"Two" represents the whole number part, which is 2.

"and" indicates the position of the decimal point.

"five tenths" means there is a 5 in the tenths place (the first digit after the decimal point). This is represented as .5.


Combining the whole number part and the decimal part, we get 2.5.


The decimal number for "two and five tenths" is $\mathbf{2.5}$.

Question 2. In the decimal $14.73$, what is the place value of the digit $7$?

Answer:

In the decimal number $14.73$, the digits have place values relative to the decimal point.

  • Digits to the left of the decimal point are the whole number part:
    • 4 is in the Ones place.
    • 1 is in the Tens place.

  • Digits to the right of the decimal point are the fractional part:
    • The first digit after the decimal point is 7. This is in the tenths place. Its value is $\frac{7}{10}$.
    • The second digit after the decimal point is 3. This is in the hundredths place. Its value is $\frac{3}{100}$.

The digit 7 is the first digit to the right of the decimal point.

Its place value is the Tenths place.

Question 3. Write the decimal $0.8$ as a fraction in its simplest form.

Answer:

To write a decimal as a fraction, we can write it as a fraction with the decimal digits as the numerator and a power of 10 as the denominator. The power of 10 is determined by the number of decimal places.


In the decimal $0.8$, there is one decimal place. So, the denominator will be $10^1 = 10$. The numerator is the decimal number without the decimal point, which is 8.


So, we can write $0.8$ as $\frac{8}{10}$.


Now, we need to simplify this fraction to its simplest form. This means dividing both the numerator and the denominator by their greatest common divisor (GCD).


Let's find the prime factorization of 8 and 10.

$\begin{array}{c|cc} 2 & 8 \\ \hline 2 & 4 \\ \hline 2 & 2 \\ \hline & 1 \end{array}$ $\begin{array}{c|cc} 2 & 10 \\ \hline 5 & 5 \\ \hline & 1 \end{array}$


The prime factors of 8 are $2 \times 2 \times 2$. The prime factors of 10 are $2 \times 5$.


The common prime factor is 2. The GCD of 8 and 10 is 2.


Now, we divide both the numerator and the denominator by the GCD, which is 2.

$\frac{\cancel{8}^{4}}{\cancel{10}_{5}}$


The simplified fraction is $\frac{4}{5}$.


Thus, the decimal $0.8$ written as a fraction in its simplest form is $\frac{4}{5}$.

Question 4. Write the fraction $\frac{17}{100}$ as a decimal number.

Answer:

To write a fraction as a decimal number, especially when the denominator is a power of 10, we can observe the number of zeros in the denominator.


The given fraction is $\frac{17}{100}$.


The denominator is 100, which has two zeros. This means the decimal number will have two decimal places.


We write the numerator, which is 17, and then place the decimal point two places from the right.


Starting from the right of 17 (which is 17.0), we move the decimal point two places to the left:

17. $\to$ 1.7 $\to$ 0.17


Alternatively, we can perform the division of 17 by 100.

$17 \div 100 = 0.17$


Thus, the fraction $\frac{17}{100}$ written as a decimal number is $0.17$.

Question 5. Represent the decimal $0.5$ on a number line.

Answer:

To represent the decimal $0.5$ on a number line, we first understand its value. The decimal $0.5$ means five-tenths, which is equivalent to the fraction $\frac{5}{10}$ or $\frac{1}{2}$.


Since $0.5$ is greater than 0 and less than 1, it lies between the integers 0 and 1 on the number line.


To represent $0.5$ specifically, we can divide the segment of the number line between 0 and 1 into equal parts.


Since $0.5 = \frac{5}{10}$, we can divide the segment between 0 and 1 into 10 equal parts. Each part represents $0.1$. We then count 5 parts from 0.


Alternatively, since $0.5 = \frac{1}{2}$, we can divide the segment between 0 and 1 into 2 equal parts. The point that divides the segment into two equal halves represents $0.5$.


Here is a representation of $0.5$ on a number line:

0---------0.5---------1

(Note: The markings are illustrative. $0.5$ is exactly in the middle of 0 and 1.)

Question 6. Convert the fraction $\frac{3}{4}$ into a decimal.

Answer:

To convert a fraction into a decimal, we can perform the division of the numerator by the denominator.


The given fraction is $\frac{3}{4}$. We need to divide 3 by 4.


Method 1: Direct Division

We divide 3 by 4. Since 3 is less than 4, we add a decimal point and zeros to the right of 3.

$\begin{array}{r} 0.75 \\ 4{\overline{\smash{\big)}\,3.00\phantom{)}}} \\ \underline{-~\phantom{(}2~8\phantom{0)}} \\ 0~20\phantom{)} \\ \underline{-~\phantom{()}(20)} \\ 0\phantom{)} \end{array}$


So, $3 \div 4 = 0.75$.


Method 2: Convert the denominator to a power of 10

We look for a number that we can multiply the denominator (4) by to get 10, 100, 1000, etc.

We can multiply 4 by 25 to get 100.

$4 \times 25 = 100$


We must multiply both the numerator and the denominator by the same number (25) to keep the value of the fraction the same.

$\frac{3}{4} = \frac{3 \times 25}{4 \times 25} = \frac{75}{100}$


Now, we convert the fraction $\frac{75}{100}$ to a decimal. The denominator 100 has two zeros, so the decimal number will have two decimal places.

We write the numerator, 75, and place the decimal point two places from the right.

75. $\to$ 7.5 $\to$ 0.75


Both methods give the same result.


Thus, the fraction $\frac{3}{4}$ converted into a decimal is $0.75$.

Question 7. What are like decimals? Give an example.

Answer:

Like decimals are decimal numbers that have the same number of decimal places.


For example, the decimal numbers $0.25$, $1.78$, and $10.05$ are like decimals because each of them has two decimal places.


In contrast, unlike decimals are decimal numbers that have a different number of decimal places. For instance, $0.5$ (one decimal place), $2.13$ (two decimal places), and $7.406$ (three decimal places) are unlike decimals.

Question 8. Convert the unlike decimals $3.4$ and $3.45$ into like decimals.

Answer:

Unlike decimals are decimal numbers that have a different number of decimal places. In this question, $3.4$ has one decimal place and $3.45$ has two decimal places, so they are unlike decimals.


Like decimals are decimal numbers that have the same number of decimal places.


To convert unlike decimals into like decimals, we find the maximum number of decimal places among the given decimals. Then, we add trailing zeros to the decimal numbers with fewer decimal places so that all numbers have the maximum number of decimal places.


Given decimals are $3.4$ and $3.45$.


The decimal $3.4$ has 1 decimal place.

The decimal $3.45$ has 2 decimal places.


The maximum number of decimal places is 2.


To convert $3.4$ to a decimal with 2 decimal places, we add one zero after the digit 4.

$3.4$ becomes $3.40$.


The decimal $3.45$ already has 2 decimal places, so it remains $3.45$.


Now, both $3.40$ and $3.45$ have 2 decimal places.


Thus, the unlike decimals $3.4$ and $3.45$ converted into like decimals are $3.40$ and $3.45$.

Question 9. Which is greater: $0.09$ or $0.1$?

Answer:

To compare decimal numbers, we can follow these steps:

1. Convert the given decimals into like decimals (decimals with the same number of decimal places).

2. Compare the whole number parts. The decimal with the greater whole number part is greater.

3. If the whole number parts are the same, compare the decimal parts digit by digit, starting from the tenths place.


The given decimal numbers are $0.09$ and $0.1$.


Step 1: Convert to like decimals.

$0.09$ has two decimal places.

$0.1$ has one decimal place.

To make them like decimals, we add a zero to $0.1$ so it has two decimal places.

$0.1$ becomes $0.10$.

Now we compare $0.09$ and $0.10$. Both have two decimal places.


Step 2: Compare the whole number parts.

The whole number part of $0.09$ is 0.

The whole number part of $0.10$ is 0.

Since the whole number parts are the same, we move to the next step.


Step 3: Compare the decimal parts digit by digit from left to right.

Compare the digits in the tenths place:

In $0.09$, the digit in the tenths place is 0.

In $0.10$, the digit in the tenths place is 1.


Since $1 > 0$, the number with 1 in the tenths place is greater.

Therefore, $0.10$ is greater than $0.09$.


Since $0.10$ is equivalent to $0.1$, this means $0.1$ is greater than $0.09$.


Thus, the greater number is $0.1$.

Question 10. Add: $5.25 + 3.70$.

Answer:

To add decimal numbers, we need to arrange the numbers vertically such that the decimal points are aligned in a straight column. Then, we add the numbers as if they were whole numbers, and place the decimal point in the sum directly below the decimal points of the numbers being added.


The given numbers are $5.25$ and $3.70$. Both numbers already have the same number of decimal places (two), so they are like decimals. We can arrange them for addition:


5.25
+ 3.70
------


Now, we add the numbers column by column from right to left:

Add the hundredths column: $5 + 0 = 5$.

Add the tenths column: $2 + 7 = 9$.

Place the decimal point.

Add the ones column: $5 + 3 = 8$.


So, the sum is 8.95.

5.25
+ 3.70
------
8.95


Thus, the sum of $5.25 + 3.70$ is $8.95$.

Question 11. Subtract: $7.8 - 4.3$.

Answer:

To subtract decimal numbers, we need to arrange the numbers vertically such that the decimal points are aligned in a straight column. We subtract the numbers as if they were whole numbers and place the decimal point in the difference directly below the decimal points of the numbers being subtracted.


The given numbers are $7.8$ and $4.3$. Both numbers already have the same number of decimal places (one). We arrange them for subtraction:


7.8
- 4.3
------


Now, we subtract the numbers column by column from right to left:

Subtract the tenths column: $8 - 3 = 5$.

Place the decimal point.

Subtract the ones column: $7 - 4 = 3$.


So, the difference is 3.5.

7.8
- 4.3
------
3.5


Thus, the difference of $7.8 - 4.3$ is $3.5$.

Question 12. Add: $2.5 + 1.35$.

Answer:

To add decimal numbers, we need to align the decimal points vertically. This ensures that we add digits with the same place value (tenths with tenths, hundredths with hundredths, and so on).


The given numbers are $2.5$ and $1.35$.


To make the addition easier and avoid errors, we can convert the numbers into like decimals, meaning they have the same number of decimal places. The number $1.35$ has two decimal places, while $2.5$ has one.


We can add a trailing zero to $2.5$ without changing its value, making it $2.50$. Now both numbers have two decimal places.

We need to add $2.50$ and $1.35$.


We arrange the numbers vertically, aligning the decimal points:

2.50
+ 1.35
------


Now, we add the numbers column by column, starting from the rightmost column (the hundredths place):

Hundredths column: $0 + 5 = 5$. Write down 5.

Tenths column: $5 + 3 = 8$. Write down 8.

Place the decimal point directly below the decimal points above.

Ones column: $2 + 1 = 3$. Write down 3.


So, the sum is 3.85.

2.50
+ 1.35
------
3.85


Thus, the sum of $2.5 + 1.35$ is $3.85$.

Question 13. Subtract: $10.0 - 6.5$.

Answer:

We are asked to subtract $6.5$ from $10.0$.


We can write the numbers vertically, aligning the decimal points, and subtract like usual whole numbers, carrying over from the left when necessary.


Let's perform the subtraction:

$\begin{array}{cc} & 10. & 0 \\ - & 6. & 5 \\ \hline & 3. & 5 \\ \hline \end{array}$


Starting from the rightmost digit:

We have $0$ in the tenths place of $10.0$ and $5$ in the tenths place of $6.5$. We cannot subtract $5$ from $0$.

We borrow from the ones place (which is $0$). The ones place needs to borrow from the tens place ($1$). The $1$ in the tens place becomes $0$. The $0$ in the ones place becomes $10$. Now, the $0$ in the tenths place borrows from the $10$ in the ones place. The $10$ in the ones place becomes $9$, and the $0$ in the tenths place becomes $10$.

Now, subtract $5$ from $10$ in the tenths place: $10 - 5 = 5$. Write down $5$ in the tenths place of the result.

Move to the ones place. We now have $9$ in the ones place of the first number (after borrowing) and $6$ in the ones place of the second number. Subtract $6$ from $9$: $9 - 6 = 3$. Write down $3$ in the ones place of the result.

The decimal point is aligned.

The tens place of the first number is now $0$. There is nothing in the tens place of the second number. $0 - 0 = 0$.

The result is $3.5$.


Thus, $10.0 - 6.5 = \textbf{3.5}$.

Question 14. Multiply: $3.2 \times 4$.

Answer:

We are asked to find the product of $3.2$ and $4$.


To multiply a decimal number by a whole number, we can ignore the decimal point initially and multiply the numbers as if they were whole numbers. Then, we place the decimal point in the product such that the number of decimal places in the product is equal to the total number of decimal places in the numbers being multiplied.


First, let's multiply $32$ by $4$ as if they were whole numbers.

$\begin{array}{cc}& & 3 & 2 \\ \times & & & 4 \\ \hline & 1 & 2 & 8 \\ \hline \end{array}$


Now, we need to place the decimal point in the product $128$.

The number $3.2$ has one decimal place (the digit $2$).

The number $4$ has zero decimal places (it is a whole number).

The total number of decimal places in the numbers being multiplied ($3.2$ and $4$) is $1 + 0 = 1$.

Therefore, the product $128$ must have one decimal place. Starting from the right and counting one place to the left, we place the decimal point between $2$ and $8$.

The result is $12.8$.


Thus, $3.2 \times 4 = \textbf{12.8}$.

Question 15. Multiply: $0.7 \times 100$.

Answer:

We are asked to find the product of $0.7$ and $100$.


When a decimal number is multiplied by $100$, the decimal point is shifted two places to the right.


Let's apply this rule to $0.7$.

The number is $0.7$. We need to move the decimal point two places to the right.

Starting with $0.7$, moving the decimal one place to the right gives $7$. To move it another place to the right, we need to add a zero to the right of $7$.

So, $0.7$ becomes $70$.


Alternatively, we can think of $0.7$ as $\frac{7}{10}$.

Then, the multiplication is $0.7 \times 100 = \frac{7}{10} \times 100$.

$\frac{7}{\cancel{10}_1} \times \cancel{100}^{10} = 7 \times 10 = 70$.


Thus, $0.7 \times 100 = \textbf{70}$.

Question 16. Divide: $15.6 \div 3$.

Answer:

We are asked to divide $15.6$ by $3$.


To divide a decimal number by a whole number, we perform the division as we would with whole numbers. The decimal point in the quotient is placed directly above the decimal point in the dividend when we reach the decimal point during the division process.


Let's perform the division of $15.6$ by $3$.

We can perform long division:

$\begin{array}{r} 5.2\phantom{)} \\ 3{\overline{\smash{\big)}\,15.6\phantom{)}}} \\ \underline{-~\phantom{(}(15)\phantom{.6)}} \\ 00.6\phantom{)} \\ \underline{-~\phantom{()}(0.6)} \\ 0\phantom{)} \end{array}$


Divide $15$ by $3$. $3 \times 5 = 15$. Write $5$ in the quotient above the $5$ of $15$. Subtract $15$ from $15$, which gives $0$.

Bring down the next digit, which is $6$. Before bringing down $6$, we cross the decimal point in the dividend ($15.6$). So, we place the decimal point in the quotient directly above the decimal point in the dividend.

Now we have $0.6$. We divide $0.6$ by $3$. $3 \times 0.2 = 0.6$. Write $2$ in the quotient after the decimal point, above the $6$. Subtract $0.6$ from $0.6$, which gives $0$.

The division is complete as the remainder is $0$.

The quotient is $5.2$.


Thus, $15.6 \div 3 = \textbf{5.2}$.

Question 17. Divide: $24.5 \div 10$.

Answer:

We are asked to divide $24.5$ by $10$.


When a decimal number is divided by $10$, the decimal point is shifted one place to the left.


Let's apply this rule to $24.5$.

The number is $24.5$. The decimal point is currently between the digits $4$ and $5$.

To divide by $10$, we move the decimal point one place to the left.

Starting with $24.5$, moving the decimal point one place to the left places it between the digits $2$ and $4$.

The result is $2.45$.


Thus, $24.5 \div 10 = \textbf{2.45}$.

Question 18. Write $\textsf{₹}5.50$ in words.

Answer:

The given amount is $\textsf{₹}5.50$. This amount represents a value in Indian Rupees and Paise.


The number before the decimal point represents the number of Rupees, and the number after the decimal point represents the number of Paise.

In $\textsf{₹}5.50$, the number before the decimal point is $5$. This means there are $5$ Rupees.

The number after the decimal point is $50$. This means there are $50$ Paise.


To write $\textsf{₹}5.50$ in words, we write the Rupees part followed by "Rupees and" and then the Paise part followed by "Paise".

$5$ in words is "Five".

$50$ in words is "Fifty".


Combining these, $\textsf{₹}5.50$ in words is:

Five Rupees and Fifty Paise.

Question 19. Write $20$ rupees and $75$ paise as a decimal in rupees.

Answer:

We are asked to express $20$ rupees and $75$ paise as a decimal amount in rupees.


In the Indian currency system, there are $100$ paise in $1$ rupee.

This means that $1$ paise is equal to $\frac{1}{100}$ of a rupee.


To convert paise into rupees, we divide the number of paise by $100$.

$75$ paise can be converted to rupees by dividing $75$ by $100$.

$75$ paise $= \frac{75}{100}$ rupees.

As a decimal, $\frac{75}{100} = 0.75$.

So, $75$ paise $= 0.75$ rupees.


Now we need to combine the rupees and the converted paise.

$20$ rupees and $75$ paise $= 20$ rupees $+ 0.75$ rupees.

$20 + 0.75 = 20.75$.


Therefore, $20$ rupees and $75$ paise written as a decimal in rupees is $\textsf{₹}20.75$.


The amount is $\textbf{$\textsf{₹}20.75$}$.

Question 20. Convert $85$ cm into meters using decimals.

Answer:

We are asked to convert a length given in centimeters ($85$ cm) into meters using decimals.


We know that the relationship between meters and centimeters is:

$1$ meter $= 100$ centimeters.


From this relationship, we can find how many meters are in $1$ centimeter:

$1$ centimeter $= \frac{1}{100}$ meter.


To convert a length from centimeters to meters, we need to divide the number of centimeters by $100$.

Given length is $85$ cm.

So, $85$ cm $= \frac{85}{100}$ meters.


Now, we need to express $\frac{85}{100}$ as a decimal.

Dividing by $100$ shifts the decimal point two places to the left. The number $85$ can be written as $85.0$.

Shifting the decimal point two places to the left in $85.0$ gives $0.850$, which is equal to $0.85$.

So, $\frac{85}{100} = 0.85$.


Therefore, $85$ cm is equal to $0.85$ meters.


The answer is $\textbf{0.85 meters}$.

Question 21. Convert $3$ kg $250$ g into kilograms using decimals.

Answer:

We are asked to convert a weight given as $3$ kilograms and $250$ grams into a single decimal value in kilograms.


We know that the relationship between kilograms and grams is:

$1$ kilogram $= 1000$ grams.


This means that $1$ gram is equal to $\frac{1}{1000}$ of a kilogram.


To convert a weight from grams to kilograms, we need to divide the number of grams by $1000$.

Given grams are $250$ g.

So, $250$ g $= \frac{250}{1000}$ kilograms.


Now, we need to express $\frac{250}{1000}$ as a decimal.

Dividing by $1000$ shifts the decimal point three places to the left. The number $250$ can be written as $250.0$.

Shifting the decimal point three places to the left in $250.0$ gives $0.250$, which is equal to $0.25$.

So, $\frac{250}{1000} = 0.250 = 0.25$ kilograms.


Now we need to combine the kilograms and the converted grams in kilograms.

$3$ kg $250$ g $= 3$ kg $+ 0.25$ kg.

$3 + 0.25 = 3.25$.


Therefore, $3$ kg $250$ g is equal to $3.25$ kilograms.


The answer is $\textbf{3.25 kilograms}$.

Question 22. Write the expanded form of the decimal $45.67$.

Answer:

We are asked to write the expanded form of the decimal number $45.67$.


In a decimal number, each digit has a place value. The place values to the left of the decimal point are units (or ones), tens, hundreds, and so on, which are powers of $10$. The place values to the right of the decimal point are tenths, hundredths, thousandths, and so on, which are reciprocal powers of $10$ (i.e., $\frac{1}{10}, \frac{1}{100}, \frac{1}{1000}$, etc.).


Let's identify the place value of each digit in the number $45.67$:

The digit $4$ is in the tens place. Its value is $4 \times 10 = 40$.

The digit $5$ is in the ones place. Its value is $5 \times 1 = 5$.

The digit $6$ is in the tenths place. Its value is $6 \times \frac{1}{10} = 0.6$.

The digit $7$ is in the hundredths place. Its value is $7 \times \frac{1}{100} = 0.07$.


The expanded form of a number is the sum of the values of its digits.

So, the expanded form of $45.67$ is the sum of the values we found:

$45.67 = 40 + 5 + 0.6 + 0.07$


We can also write the decimal parts as fractions:

$45.67 = 4 \times 10 + 5 \times 1 + 6 \times \frac{1}{10} + 7 \times \frac{1}{100}$

or

$45.67 = 4 \times 10 + 5 \times 1 + 6 \times 10^{-1} + 7 \times 10^{-2}$


The expanded form of $45.67$ is $\textbf{40 + 5 + 0.6 + 0.07}$.

Question 23. What is the place value of the digit $0$ in the decimal $1.05$?

Answer:

We are asked to find the place value of the digit $0$ in the decimal number $1.05$.


In a decimal number, the positions of the digits relative to the decimal point determine their place values.

Starting from the decimal point and moving to the left, the place values are ones, tens, hundreds, and so on.

Starting from the decimal point and moving to the right, the place values are tenths, hundredths, thousandths, and so on.


Let's look at the number $1.05$:

  • The digit $1$ is to the left of the decimal point, in the ones place.
  • The digit $0$ is the first digit to the right of the decimal point.
  • The digit $5$ is the second digit to the right of the decimal point.

The first place to the right of the decimal point is the \textbf{tenths place}.

Since the digit $0$ is in the first position to the right of the decimal point, its place value is the tenths place.


The value of the digit $0$ in the tenths place is $0 \times \frac{1}{10} = 0$.


The place value of the digit $0$ in $1.05$ is the \textbf{tenths} place.

Question 24. Which is smaller: $1.99$ or $2.01$?

Answer:

We are asked to compare the two decimal numbers, $1.99$ and $2.01$, and determine which one is smaller.


To compare decimal numbers, we can compare the digits from left to right, starting with the whole number part.


First, let's compare the whole number parts of the two numbers:

  • The whole number part of $1.99$ is $1$ (the digit to the left of the decimal point).
  • The whole number part of $2.01$ is $2$ (the digit to the left of the decimal point).

Now, we compare the whole numbers $1$ and $2$.

We know that $1$ is less than $2$.

$1 < 2$


When comparing decimal numbers, if the whole number parts are different, the number with the smaller whole number part is the smaller number, regardless of the digits after the decimal point.

Since the whole number part of $1.99$ ($1$) is smaller than the whole number part of $2.01$ ($2$), the number $1.99$ is smaller than $2.01$.


Thus, the smaller number is $\textbf{1.99}$.

Question 25. Calculate $1.5 \times 0.3$.

Answer:

We are asked to calculate the product of $1.5$ and $0.3$.


To multiply decimal numbers, we can multiply them as if they were whole numbers. Then, we count the total number of digits after the decimal point in both the numbers being multiplied. The product will have the same number of digits after the decimal point.


Let's ignore the decimal points for now and multiply $15$ by $3$.

$\begin{array}{cc}& & 1 & 5 \\ \times & & & 3 \\ \hline & & 4 & 5 \\ \hline \end{array}$

The product of $15$ and $3$ is $45$.


Now, let's count the total number of decimal places in the original numbers:

  • The number $1.5$ has one digit after the decimal point ($5$).
  • The number $0.3$ has one digit after the decimal point ($3$).

The total number of decimal places is $1 + 1 = 2$.


We need to place the decimal point in the product $45$ such that there are exactly two digits after the decimal point. Starting from the rightmost digit of $45$, we move the decimal point two places to the left.

If we consider $45$ as $45.0$, moving the decimal two places left gives $0.45$.


Therefore, $1.5 \times 0.3 = 0.45$.


The result is $\textbf{0.45}$.



Long Answer Type Questions

Question 1. Explain the decimal place value system. Write the number $537.189$ in expanded form and in words. Identify the place value of each digit in the number.

Answer:

The decimal place value system is a system of assigning a value to a digit based on its position relative to the decimal point. It is a base-10 system, meaning that each place value is ten times larger than the place value to its right.

For digits to the left of the decimal point, the place values are powers of 10, starting from $10^0$ (Ones place) just to the left of the decimal, then $10^1$ (Tens place), $10^2$ (Hundreds place), and so on, moving leftwards.

For digits to the right of the decimal point, the place values are negative powers of 10, starting from $10^{-1}$ (Tenths place) just to the right of the decimal, then $10^{-2}$ (Hundredths place), $10^{-3}$ (Thousandths place), and so on, moving rightwards.


The given number is $537.189$.


Place Value of each digit in $537.189$:

Starting from the leftmost digit:

The digit 5 is in the Hundreds place. Its value is $5 \times 100 = 500$.

The digit 3 is in the Tens place. Its value is $3 \times 10 = 30$.

The digit 7 is in the Ones place. Its value is $7 \times 1 = 7$.

The digit 1 is in the Tenths place. Its value is $1 \times 0.1 = 0.1$.

The digit 8 is in the Hundredths place. Its value is $8 \times 0.01 = 0.08$.

The digit 9 is in the Thousandths place. Its value is $9 \times 0.001 = 0.009$.


Expanded Form of $537.189$:

The number can be written as the sum of the values of each digit based on its place value.

$537.189 = 5 \times 100 + 3 \times 10 + 7 \times 1 + 1 \times 0.1 + 8 \times 0.01 + 9 \times 0.001$

Using powers of 10, the expanded form is:

$537.189 = 5 \times 10^2 + 3 \times 10^1 + 7 \times 10^0 + 1 \times 10^{-1} + 8 \times 10^{-2} + 9 \times 10^{-3}$


In Words:

The number $537.189$ in words is:

Five hundred thirty-seven and one hundred eighty-nine thousandths.

Question 2. Explain how to represent decimal numbers on a number line. Illustrate by representing $0.2, 0.8,$ and $1.5$ on a number line. Describe the steps involved in locating $1.5$ on the number line.

Answer:

Representing decimal numbers on a number line involves dividing the unit intervals (the space between consecutive integers like 0 and 1, 1 and 2, etc.) into smaller, equal parts based on the decimal places.

To represent a decimal like $0.2$ or $0.8$ (which have one decimal place, representing tenths), we divide the unit interval from 0 to 1 into 10 equal parts. Each part represents $0.1$ (one-tenth). Starting from 0, the first mark is $0.1$, the second is $0.2$, and so on, up to $0.9$, after which comes 1.

For decimal numbers greater than 1, like $1.5$, we first locate the integer part (which is 1). Then, we consider the unit interval starting from that integer (the interval from 1 to 2). We again divide this interval into 10 equal parts, each representing $0.1$. Starting from 1, the first mark is $1.1$, the second is $1.2$, and so on.


Illustration:

Consider a number line. Mark the integers 0, 1, and 2.

Divide the segment between 0 and 1 into 10 equal smaller segments. These marks will represent 0.1, 0.2, 0.3, ..., 0.9.

Divide the segment between 1 and 2 into 10 equal smaller segments. These marks will represent 1.1, 1.2, 1.3, ..., 1.9.

To represent $0.2$, locate the second mark to the right of 0 in the interval [0, 1].

To represent $0.8$, locate the eighth mark to the right of 0 in the interval [0, 1].

To represent $1.5$, locate the fifth mark to the right of 1 in the interval [1, 2].

A visual representation would look something like this (numbers are marked below the line, and dots/points above):

$... \quad -1 \quad \quad 0 \quad \cdot \quad \cdot \quad 0.2 \quad \cdot \quad \cdot \quad \cdot \quad \cdot \quad 0.8 \quad \cdot \quad 1 \quad \cdot \quad \cdot \quad \cdot \quad \cdot \quad 1.5 \quad \cdot \quad \cdot \quad \cdot \quad \cdot \quad 2 \quad \quad ...$


Steps to locate $1.5$ on the number line:

Here are the steps involved:

1. Draw a number line and mark the integers (e.g., 0, 1, 2, 3, ...).

2. Identify the two consecutive integers between which the decimal number lies. Since $1 < 1.5 < 2$, the number $1.5$ lies between the integers 1 and 2.

3. Focus on the unit interval between 1 and 2. Divide this segment into 10 equal parts. Each part represents $0.1$.

4. Starting from the integer 1, move to the right along these subdivisions. The first mark is $1 + 0.1 = 1.1$, the second is $1 + 0.2 = 1.2$, the third is $1 + 0.3 = 1.3$, the fourth is $1 + 0.4 = 1.4$, and the fifth mark is $1 + 0.5 = 1.5$.

5. The point corresponding to the fifth mark to the right of 1 represents $1.5$ on the number line.

Question 3. Explain the process of converting fractions to decimals and decimals to fractions. Convert $\frac{4}{5}$ and $\frac{7}{8}$ into decimals. Convert $0.625$ and $2.4$ into fractions and reduce them to their simplest forms.

Answer:

Converting Fractions to Decimals:

To convert a fraction to a decimal, you perform division. The numerator is divided by the denominator. The result of this division is the decimal representation of the fraction. If the denominator is a power of 10 (like 10, 100, 1000, etc.) or can be easily converted to a power of 10 by multiplying the numerator and denominator by the same number, you can also directly write the decimal by placing the decimal point according to the number of zeros in the denominator.


Converting $\frac{4}{5}$ to a decimal:

Method 1: Division

Divide 4 by 5:

$\begin{array}{r} 0.8 \\ 5{\overline{\smash{\big)}\,4.0}} \\ \underline{-~\phantom{(}(40)} \\ 0\phantom{)} \end{array}$

So, $\frac{4}{5} = 0.8$.

Method 2: Making the denominator a power of 10

Multiply the numerator and denominator by 2 to make the denominator 10:

$\frac{4}{5} = \frac{4 \times 2}{5 \times 2} = \frac{8}{10}$.

Since the denominator is 10 (one zero), the decimal point is one place from the right.

$\frac{8}{10} = 0.8$.


Converting $\frac{7}{8}$ to a decimal:

Method 1: Division

Divide 7 by 8:

$\begin{array}{r} 0.875 \\ 8{\overline{\smash{\big)}\,7.000}} \\ \underline{-~\phantom{(}(64)}\downarrow\phantom{00)} \\ 60\phantom{0)} \\ \underline{-~\phantom{()}(56)}\downarrow\phantom{0)} \\ 40\phantom{)} \\ \underline{-~\phantom{()}(40)} \\ 0\phantom{)} \end{array}$

So, $\frac{7}{8} = 0.875$.

Method 2: Making the denominator a power of 1000

Multiply the numerator and denominator by 125 to make the denominator 1000 ($8 \times 125 = 1000$).

$\frac{7}{8} = \frac{7 \times 125}{8 \times 125} = \frac{875}{1000}$.

Since the denominator is 1000 (three zeros), the decimal point is three places from the right.

$\frac{875}{1000} = 0.875$.


Converting Decimals to Fractions:

To convert a decimal to a fraction, write the decimal number as a fraction where the numerator is the decimal number without the decimal point, and the denominator is a power of 10. The power of 10 in the denominator is determined by the number of digits after the decimal point. If there is one digit after the decimal, the denominator is 10 ($10^1$). If there are two digits, the denominator is 100 ($10^2$), if three digits, it's 1000 ($10^3$), and so on.

After writing the fraction, simplify it to its simplest form by dividing both the numerator and the denominator by their greatest common divisor (GCD).


Converting $0.625$ to a fraction:

The number $0.625$ has three digits after the decimal point (6, 2, and 5).

So, the denominator will be $10^3 = 1000$. The numerator is the number without the decimal point, which is 625.

$0.625 = \frac{625}{1000}$.

Now, simplify the fraction $\frac{625}{1000}$. We can divide both numerator and denominator by common factors. Both are divisible by 5, 25, or even 125.

Using 125 (which is the GCD):

$\frac{625 \div 125}{1000 \div 125} = \frac{5}{8}$.

Using step-by-step division:

$\frac{\cancel{625}^{125}}{\cancel{1000}_{200}} = \frac{\cancel{125}^{25}}{\cancel{200}_{40}} = \frac{\cancel{25}^{5}}{\cancel{40}_{8}} = \frac{5}{8}$.

The simplest form of $0.625$ as a fraction is $\frac{5}{8}$.


Converting $2.4$ to a fraction:

The number $2.4$ has a whole number part (2) and a decimal part (0.4).

Method 1: Convert the decimal part to a fraction and add it to the whole number.

The decimal part is $0.4$. It has one digit after the decimal point.

$0.4 = \frac{4}{10}$.

Simplify $\frac{4}{10}$ by dividing numerator and denominator by their GCD, which is 2:

$\frac{4 \div 2}{10 \div 2} = \frac{2}{5}$.

Now add the whole number part (2) to the fraction $\frac{2}{5}$:

$2 + \frac{2}{5} = \frac{2 \times 5}{5} + \frac{2}{5} = \frac{10}{5} + \frac{2}{5} = \frac{10+2}{5} = \frac{12}{5}$.

Method 2: Treat the entire number as a decimal and write it over a power of 10.

$2.4$ has one digit after the decimal point.

$2.4 = \frac{24}{10}$.

Now, simplify the fraction $\frac{24}{10}$ by dividing both numerator and denominator by their GCD, which is 2:

$\frac{24 \div 2}{10 \div 2} = \frac{12}{5}$.

The simplest form of $2.4$ as a fraction is $\frac{12}{5}$. This is an improper fraction. If needed, it can be written as a mixed number $2\frac{2}{5}$.

Question 4. Define like and unlike decimals. Explain how to convert unlike decimals into like decimals. Arrange the following decimals in ascending order: $3.5, 3.45, 3.05, 3.405$. Show the steps you used for comparison.

Answer:

Like Decimals:

Like decimals are decimal numbers that have the same number of digits after the decimal point. For example, $2.56$, $10.00$, and $0.78$ are like decimals because each has two digits after the decimal point.


Unlike Decimals:

Unlike decimals are decimal numbers that have a different number of digits after the decimal point. For example, $3.4$, $5.12$, and $0.675$ are unlike decimals because they have one, two, and three digits after the decimal point, respectively.


Converting Unlike Decimals into Like Decimals:

To convert unlike decimals into like decimals, we add trailing zeros to the right of the last digit after the decimal point. The number of zeros added to each decimal is such that all decimals end up having the same number of decimal places as the decimal with the maximum number of decimal places among the given numbers. Adding trailing zeros after the last decimal digit does not change the value of the decimal number.


Arranging $3.5, 3.45, 3.05, 3.405$ in ascending order:

The given decimals are: $3.5, 3.45, 3.05, 3.405$.

Step 1: Count the number of decimal places for each number.

$3.5$ has 1 decimal place.

$3.45$ has 2 decimal places.

$3.05$ has 2 decimal places.

$3.405$ has 3 decimal places.

Step 2: Identify the maximum number of decimal places. The maximum number of decimal places is 3.

Step 3: Convert all the given decimals into like decimals by adding trailing zeros so that each number has 3 decimal places.

$3.5 = 3.500$ (added two zeros)

$3.45 = 3.450$ (added one zero)

$3.05 = 3.050$ (added one zero)

$3.405 = 3.405$ (already has 3 decimal places)

The like decimals are: $3.500, 3.450, 3.050, 3.405$.

Step 4: Compare the like decimals. We compare the whole number parts first. All numbers have 3 as the whole number part. Then, we compare the digits in the tenths place, then the hundredths place, and then the thousandths place.

Comparing the tenths place digits (5, 4, 0, 4):

The smallest tenths digit is 0 (in 3.050).

The other tenths digits are 5 and 4. The smallest of these is 4 (in 3.450 and 3.405).

The largest tenths digit is 5 (in 3.500).

So, $3.050$ is the smallest number.

Next, compare the numbers with tenths digit 4: $3.450$ and $3.405$.

Compare the hundredths place digits (5 in 3.450 and 0 in 3.405):

$0 < 5$, so $3.405$ is smaller than $3.450$.

Finally, $3.500$ is the largest among the four numbers.

Arranging the like decimals in ascending order: $3.050, 3.405, 3.450, 3.500$.

Step 5: Write the original decimals in the order determined in Step 4.

The original decimals in ascending order are: $3.05, 3.405, 3.45, 3.5$.

Question 5. Explain the method for adding decimal numbers. Calculate the sum of $18.5, 2.76,$ and $0.935$. Show the calculation steps, including aligning the decimal points.

Answer:

Method for Adding Decimal Numbers:

The method for adding decimal numbers is similar to adding whole numbers, with the crucial step of aligning the decimal points. Here are the steps:

1. Write the numbers vertically, one below the other, ensuring that the decimal points are perfectly aligned in a straight column.

2. If the numbers have a different number of decimal places (unlike decimals), you can optionally add trailing zeros to the right of the decimal point for each number so that all numbers have the same number of decimal places. This makes the column addition easier, especially for beginners, as all columns will be filled with digits.

3. Start adding the digits from the rightmost column, just as you would with whole numbers. Carry over any tens to the next column to the left.

4. Place the decimal point in the sum directly below the aligned decimal points of the numbers being added.

5. Continue adding the digits in each column, moving from right to left, including the digits to the left of the decimal point, carrying over as necessary.


Calculating the sum of $18.5, 2.76,$ and $0.935$:

The given decimal numbers are $18.5, 2.76,$ and $0.935$.

Step 1: Write the numbers vertically and align the decimal points.

$18.5$

$\phantom{0}2.76$

$\phantom{0}0.935$

Step 2 (Optional but helpful): Make them like decimals by adding trailing zeros. The maximum number of decimal places is 3 (in 0.935).

$18.500$

$\phantom{0}2.760$

$\phantom{0}0.935$

Step 3: Add the numbers column by column from right to left, starting from the thousandths place.

Adding the thousandths column (0 + 0 + 5): 5. Write down 5.

Adding the hundredths column (0 + 6 + 3): 9. Write down 9.

Adding the tenths column (5 + 7 + 9): 21. Write down 1 and carry over 2 to the ones column.

Place the decimal point in the sum.

Adding the ones column (8 + 2 + 0 + the carried over 2): $8 + 2 + 0 + 2 = 12$. Write down 2 and carry over 1 to the tens column.

Adding the tens column (1 + 0 + 0 + the carried over 1): $1 + 0 + 0 + 1 = 2$. Write down 2.

The calculation is shown below:

$\begin{array}{c@{\,}c@{}c@{}c@{}c} & 1 & 8 & . & 5 & 0 & 0 \\ & \phantom{0} & 2 & . & 7 & 6 & 0 \\ + & \phantom{0} & 0 & . & 9 & 3 & 5 \\ \hline & 2 & 2 & . & 1 & 9 & 5 \\ \hline \end{array}$

Step 4: The sum is $22.195$.


Calculation Steps with Alignment:

Write the numbers:

$\quad 18.5$

$\quad \phantom{0}2.76$

$\underline{+ \phantom{0}0.935}$

Align decimal points and add trailing zeros:

$\quad 18.500$

$\quad \phantom{0}2.760$

$\underline{+ \phantom{0}0.935}$

Perform column addition from right to left:

$\quad 1 \quad 1 \quad \quad$ (Carries)

$\quad 18.500$

$\quad \phantom{0}2.760$

$\underline{+ \phantom{0}0.935}$

$\quad 22.195$ (Sum)

The final sum is $22.195$.

Question 6. Explain the method for subtracting decimal numbers. Calculate $25.0 - 13.87$ and $40 - 15.65$. Show the calculation steps, including aligning the decimal points and using zeros where needed.

Answer:

Method for Subtracting Decimal Numbers:

Subtracting decimal numbers follows a method similar to subtracting whole numbers, with the key requirement of aligning the decimal points. Here are the steps:

1. Write the numbers vertically, with the number being subtracted (subtrahend) below the number from which it is being subtracted (minuend). Ensure that the decimal points are aligned in a straight column.

2. If the numbers have a different number of decimal places, add trailing zeros to the right of the decimal point in the minuend so that it has the same number of decimal places as the subtrahend. This is crucial because you need a digit in each place value column to subtract from.

3. Start subtracting the digits from the rightmost column. If a digit in the minuend is smaller than the corresponding digit in the subtrahend, you need to borrow from the digit in the next column to the left.

4. Place the decimal point in the difference (the result of subtraction) directly below the aligned decimal points.

5. Continue subtracting column by column, moving from right to left, including the digits to the left of the decimal point, borrowing as necessary.


Calculating $25.0 - 13.87$:

The numbers are $25.0$ and $13.87$. The number of decimal places is 1 for 25.0 and 2 for 13.87. We need to add a zero to 25.0 to make it 25.00 so both numbers have 2 decimal places.

Step 1: Write the numbers vertically and align the decimal points. Add trailing zeros.

$\quad 25.00$

$\underline{- 13.87}$

Step 2: Subtract column by column from right to left, borrowing where necessary.

Starting from the hundredths place: We need to subtract 7 from 0. We cannot do this, so we borrow from the tenths place.

The tenths place has 0. We need to borrow from the ones place (5). Borrow 1 from 5, which becomes 4. The 0 in the tenths place becomes 10. Now, borrow 1 from 10 (tenths place), which becomes 9. The 0 in the hundredths place becomes 10.

Now, subtract 7 from 10 in the hundredths column: $10 - 7 = 3$. Write down 3.

In the tenths column, we have 9 (after borrowing). Subtract 8 from 9: $9 - 8 = 1$. Write down 1.

Place the decimal point in the difference.

In the ones column, we have 4 (after borrowing). Subtract 3 from 4: $4 - 3 = 1$. Write down 1.

In the tens column, subtract 1 from 2: $2 - 1 = 1$. Write down 1.

The calculation is shown below:

$\begin{array}{c@{\,}c@{}c@{}c@{}c@{}c} & \phantom{0}\cancel{2}^{1} & \cancel{5}^{4} & . & \cancel{0}^{9} & \cancel{0}^{10} \\ - & 1 & 3 & . & 8 & 7 \\ \hline & 1 & 1 & . & 1 & 3 \\ \hline \end{array}$

The difference is $11.13$.


Calculating $40 - 15.65$:

The numbers are 40 and 15.65. 40 is a whole number. We can write it as a decimal with a decimal point and trailing zeros. The number 15.65 has 2 decimal places, so we write 40 as 40.00.

Step 1: Write the numbers vertically and align the decimal points. Add trailing zeros.

$\quad 40.00$

$\underline{- 15.65}$

Step 2: Subtract column by column from right to left, borrowing where necessary.

Starting from the hundredths place: Subtract 5 from 0. Cannot do this. Need to borrow.

The tenths place has 0. Need to borrow from the ones place (0). Need to borrow from the tens place (4).

Borrow 1 from 4 (tens place), which becomes 3. The 0 in the ones place becomes 10. Now, borrow 1 from 10 (ones place), which becomes 9. The 0 in the tenths place becomes 10. Now, borrow 1 from 10 (tenths place), which becomes 9. The 0 in the hundredths place becomes 10.

Now, subtract 5 from 10 in the hundredths column: $10 - 5 = 5$. Write down 5.

In the tenths column, we have 9 (after borrowing). Subtract 6 from 9: $9 - 6 = 3$. Write down 3.

Place the decimal point in the difference.

In the ones column, we have 9 (after borrowing). Subtract 5 from 9: $9 - 5 = 4$. Write down 4.

In the tens column, we have 3 (after borrowing). Subtract 1 from 3: $3 - 1 = 2$. Write down 2.

The calculation is shown below:

$\begin{array}{c@{\,}c@{}c@{}c@{}c@{}c} & \cancel{4}^{3} & \cancel{0}^{9} & . & \cancel{0}^{9} & \cancel{0}^{10} \\ - & 1 & 5 & . & 6 & 5 \\ \hline & 2 & 4 & . & 3 & 5 \\ \hline \end{array}$

The difference is $24.35$.

Question 7. Explain how to multiply a decimal number by a whole number and by another decimal number. Calculate $12.8 \times 5$ and $3.4 \times 2.6$. Show the steps for both calculations.

Answer:

Multiplying a Decimal Number by a Whole Number:

To multiply a decimal number by a whole number, follow these steps:

1. Ignore the decimal point in the decimal number and multiply the numbers as if they were both whole numbers.

2. Count the total number of decimal places in the original decimal number.

3. Place the decimal point in the product (the result of the multiplication) starting from the right and moving to the left, by the number of decimal places counted in step 2.


Calculating $12.8 \times 5$:

Step 1: Multiply 128 by 5, ignoring the decimal point in 12.8.

$\begin{array}{cc}& & 1 & 2 & 8 \\ \times & & & & 5 \\ \hline & & 6 & 4 & 0 \\ \hline \end{array}$

The product of 128 and 5 is 640.

Step 2: Count the number of decimal places in 12.8. There is one digit (8) after the decimal point, so there is 1 decimal place.

Step 3: Place the decimal point in the product 640. Starting from the right, move the decimal point 1 place to the left.

$640 \rightarrow 64.0$

So, $12.8 \times 5 = 64.0$ or $64$.


Multiplying a Decimal Number by Another Decimal Number:

To multiply a decimal number by another decimal number, follow these steps:

1. Ignore the decimal points in both numbers and multiply them as if they were whole numbers.

2. Count the total number of decimal places in both original decimal numbers (sum of the number of decimal places in the multiplicand and the multiplier).

3. Place the decimal point in the product. Starting from the right, move the decimal point to the left by the total number of decimal places counted in step 2.


Calculating $3.4 \times 2.6$:

Step 1: Multiply 34 by 26, ignoring the decimal points in 3.4 and 2.6.

$\begin{array}{cc}& & & 3 & 4 \\ \times & & & 2 & 6 \\ \hline && 2 & 0 & 4 \\ & 6 & 8 & \times \\ \hline & 8 & 8 & 4 \\ \hline \end{array}$

The product of 34 and 26 is 884.

Step 2: Count the total number of decimal places in 3.4 and 2.6.

3.4 has 1 decimal place.

2.6 has 1 decimal place.

Total number of decimal places = $1 + 1 = 2$ decimal places.

Step 3: Place the decimal point in the product 884. Starting from the right, move the decimal point 2 places to the left.

$884 \rightarrow 8.84$

So, $3.4 \times 2.6 = 8.84$.

Question 8. Explain how to divide a decimal number by a whole number and by another decimal number. Calculate $36.9 \div 9$ and $4.2 \div 0.6$. Show the steps for both calculations.

Answer:

Explanation of Division of Decimal Numbers

Dividing decimal numbers involves similar steps to dividing whole numbers, but with careful handling of the decimal point.


Dividing a Decimal Number by a Whole Number

To divide a decimal number by a whole number, follow these steps:

1. Perform the division as if both numbers were whole numbers.

2. Place the decimal point in the quotient directly above the decimal point in the dividend.

3. Continue the division until the remainder is zero or the desired number of decimal places is reached.


Calculation: $36.9 \div 9$

Let's divide $36.9$ by $9$ using the steps above.

Step 1: Divide the whole number part of the dividend ($36$) by the divisor ($9$).

$36 \div 9 = 4$.

Write $4$ in the quotient above the $6$ in $36.9$.

Step 2: Place the decimal point in the quotient directly above the decimal point in $36.9$. The quotient now looks like $4.$.

Step 3: Bring down the next digit from the dividend, which is $9$. We now have $9$ to divide.

Step 4: Divide the brought down digit ($9$) by the divisor ($9$).

$9 \div 9 = 1$.

Write $1$ in the quotient next to the decimal point.

The quotient is $4.1$. The remainder is $0$.

Therefore, $36.9 \div 9 = 4.1$.


Dividing a Decimal Number by Another Decimal Number

To divide a decimal number by another decimal number, follow these steps:

1. Move the decimal point in the divisor to the right until it becomes a whole number.

2. Move the decimal point in the dividend to the right by the same number of places as in the divisor. Add zeros to the dividend if necessary.

3. Divide the new dividend by the new whole number divisor as explained in the previous section (dividing a decimal by a whole number).


Calculation: $4.2 \div 0.6$

Let's divide $4.2$ by $0.6$ using the steps above.

Step 1: Move the decimal point in the divisor ($0.6$) to the right until it is a whole number. Moving it $1$ place to the right gives $6$. The divisor becomes $6$.

Step 2: Move the decimal point in the dividend ($4.2$) to the right by the same number of places ($1$ place). Moving it $1$ place to the right gives $42$. The new dividend becomes $42$.

The original division $4.2 \div 0.6$ is now equivalent to $42 \div 6$.

Step 3: Perform the division of the new dividend ($42$) by the new divisor ($6$).

$42 \div 6 = 7$.

Since the new dividend ($42$) is a whole number and the division is exact, the result is a whole number.

Therefore, $4.2 \div 0.6 = 7$.

Question 9. Rohan bought a notebook for $\textsf{₹}25.75$, a pen for $\textsf{₹}15.50$, and an eraser for $\textsf{₹}3.25$. How much total money did he spend?

Answer:

Given:

Cost of notebook = $\textsf{₹}25.75$

Cost of pen = $\textsf{₹}15.50$

Cost of eraser = $\textsf{₹}3.25$


To Find:

The total money Rohan spent.


Solution:

To find the total money Rohan spent, we need to add the costs of all the items he bought.

Total money spent = Cost of notebook + Cost of pen + Cost of eraser

Total money spent = $\textsf{₹}25.75 + \textsf{₹}15.50 + \textsf{₹}3.25$

We add these decimal numbers by aligning the decimal points and adding each column:

$\begin{array}{c} \\ \\ + \\ \hline \\ \hline \end{array} \begin{array}{cccccc} & 2 & 5 & . & 7 & 5 \\ & 1 & 5 & . & 5 & 0 \\ & & 3 & . & 2 & 5 \\ \hline & 4 & 4 & . & 5 & 0 \\ \hline \end{array}$

Adding the hundredths column ($5+0+5$) gives $10$. We write $0$ and carry over $1$ to the tenths column.

Adding the tenths column ($7+5+2+1$ carried) gives $15$. We write $5$ and carry over $1$ to the units column.

Adding the units column ($5+5+3+1$ carried) gives $14$. We write $4$ and carry over $1$ to the tens column.

Adding the tens column ($2+1+0+1$ carried) gives $4$. We write $4$.

We place the decimal point in the result, aligned with the decimal points above.

The sum is $44.50$.

Therefore, the total money Rohan spent is $\textsf{₹}44.50$.

Question 10. Priya had $\textsf{₹}500$. She bought groceries worth $\textsf{₹}345.75$. How much money is left with her?

Answer:

Given:

Total money Priya had = $\textsf{₹}500$

Cost of groceries = $\textsf{₹}345.75$


To Find:

The amount of money left with Priya.


Solution:

To find the money left, we need to subtract the cost of groceries from the total money Priya had.

Money left = Total money - Cost of groceries

Money left = $\textsf{₹}500 - \textsf{₹}345.75$

We perform the subtraction by aligning the decimal points. We can write $500$ as $500.00$ for easier subtraction with a number having two decimal places.

$\begin{array}{cccccc} & 5 & 0 & 0 & . & 0 & 0 \\ - & 3 & 4 & 5 & . & 7 & 5 \\ \hline & 1 & 5 & 4 & . & 2 & 5 \\ \hline \end{array}$

We subtract column by column from right to left, borrowing when necessary.

Subtracting the hundredths column: $0 - 5$. We need to borrow. Borrowing from the tenths place gives $10 - 5 = 5$. The tenths place becomes $9$ (from $0$ after borrowing from the units place), and the units place becomes $9$ (from $0$ after borrowing from the tens place), the tens place becomes $9$ (from $0$ after borrowing from the hundreds place), and the hundreds place becomes $4$ (from $5$).

Subtracting the tenths column: $9 - 7 = 2$.

Subtracting the units column: $9 - 5 = 4$.

Subtracting the tens column: $9 - 4 = 5$.

Subtracting the hundreds column: $4 - 3 = 1$.

The result is $154.25$.

Therefore, $\textsf{₹}154.25$ is left with Priya.

Question 11. A tailor uses $2.3$ meters of cloth to stitch one shirt. How much cloth is needed to stitch $15$ such shirts?

Answer:

Given:

Cloth needed for one shirt = $2.3$ meters

Number of shirts to be stitched = $15$


To Find:

The total amount of cloth needed to stitch $15$ shirts.


Solution:

To find the total cloth needed, we multiply the cloth needed for one shirt by the number of shirts.

Total cloth needed = (Cloth needed for one shirt) $\times$ (Number of shirts)

Total cloth needed = $2.3 \times 15$ meters

We multiply $2.3$ by $15$. We can multiply the numbers as if they were whole numbers ($23 \times 15$) and then place the decimal point in the result.

$\begin{array}{cc}& & 2 & 3 \\ \times & & 1 & 5 \\ \hline & 1 & 1 & 5 \\ & 2 & 3 & \times \\ \hline & 3 & 4 & 5 \\ \hline \end{array}$

The product of $23$ and $15$ is $345$.

Now, we need to place the decimal point in the result. The number $2.3$ has one decimal place, and the number $15$ has zero decimal places. The total number of decimal places in the product should be $1 + 0 = 1$.

So, we place the decimal point one place from the right in $345$.

The result is $34.5$.

Therefore, $2.3 \times 15 = 34.5$.

The total cloth needed to stitch $15$ shirts is $34.5$ meters.

Question 12. A wire of length $18.75$ meters is cut into $5$ equal pieces. What is the length of each piece?

Answer:

Given:

Total length of the wire = $18.75$ meters

Number of equal pieces the wire is cut into = $5$


To Find:

The length of each piece of wire.


Solution:

To find the length of each equal piece, we need to divide the total length of the wire by the number of pieces.

Length of each piece = (Total length of wire) $\div$ (Number of pieces)

Length of each piece = $18.75 \div 5$ meters

We perform the division $18.75 \div 5$. We can divide the decimal number by the whole number as follows:

1. Divide the whole number part of the dividend ($18$) by the divisor ($5$).

$18 \div 5 = 3$ with a remainder of $3$.

Write $3$ in the quotient above the $8$ in $18.75$. The decimal point in the quotient is placed directly above the decimal point in the dividend.

2. Bring down the first digit after the decimal point, which is $7$. We now have $37$ to divide.

$37 \div 5 = 7$ with a remainder of $2$.

Write $7$ in the quotient after the decimal point.

3. Bring down the next digit, which is $5$. We now have $25$ to divide.

$25 \div 5 = 5$ with a remainder of $0$.

Write $5$ in the quotient after the $7$.

The quotient is $3.75$. The remainder is $0$.

Therefore, $18.75 \div 5 = 3.75$.

The length of each piece of wire is $3.75$ meters.